Решить определитель третьего порядка онлайн: Онлайн калькулятор. Определитель матрицы. Детерминант матрицы

Определители третьего порядка с примерами | Математика

Пусть дана квадратная матрица третьего порядка

ОПРЕДЕЛЕНИЕ 1.8

Определителем третьего порядка, соответствующим данной квадратной матрице А, называется число

(1.7)

Помощь с решением задач

Определитель третьего порядка обозначается символом

(1.8)

где числа называются его элементами.

Индексы у элемента показывают номера строки и столбца, на пересечении которых записан этот элемент.

Например, элемент расположен на пересечении второй строки и третьего столбца .

Элементы образуют главную диагональ определителя, а элементы побочную диагональ.

Определение имеет сложный по форме вид, поэтому для нахождения определителя третьего порядка предложены более простые правила. Так, согласно правилу треугольников необходимо:

  1. вычислить с собственными знаками произведения элементов , лежащих на главной диагонали и в вершинах двух равнобедренных треугольников, основания которых параллельны этой диагонали ;
  2. найти произведения элементов, лежащих на побочной диагонали и в вершинах двух равнобедренных треугольников, основания которых параллельны побочной диагонали, и взять их с противоположными знаками ;
  3. найти общую сумму всех произведений.

ПРИМЕР 1.1.7

Все свойства определителей второго порядка справедливы и для определителей третьего порядка. Доказательства этих свойств основаны на вычислении определителя третьего порядка по формуле (1.7).

Например, покажем, что определитель, у которого элементы двух его строк пропорциональны, равен нулю. Действительно,

Аналогично проверяется справедливость и других свойств.

Пусть дан определитель (1.8) третьего порядка.

ОПРЕДЕЛЕНИЕ 1.9: Минором элемента , где определителя третьего порядка, называется определитель второго порядка, полученный из данного вычеркиванием й строки и го столбца. Так, например, минор элемента есть определитель

а минор элемента есть

С помощью миноров определитель (7) можно записать в виде

(1.9)

ОПРЕДЕЛЕНИЕ 1.10: Алгебраическим дополнением элемента , где , называется минор этого элемента, взятый со знаком . По определению 4.3 имеем

где /(1.10)

Например,

и т.д.

ТЕОРЕМА 1.1 Разложение определителя по элементам строки или столбца

Определитель третьего порядка равен сумме произведений элементов любой его строки (столбца) на их алгебраические дополнения. Иными словами, имеют место шесть равенств:

(1.11)

Проверим, например, справедливость равенства

Согласно определениям минора и алгебраического дополнения получим

ТЕОРЕМА 1. 2 Сумма произведений элементов какой- либо строки (столбца) определителя на алгебраические дополнения элементов любой другой его строки (столбца) равна нулю.

Для определенности выберем элементы первой строки и алгебраические дополнения элементов второй строки определителя. Составим сумму произведений и покажем, что эта сумма равна нулю.

Действительно,

Аналогично проверяется равенство нулю и всех других подобных сумм.

В заключение рассмотрим схему использования свойств определителя и теоремы разложения при вычислении определителя.

ПРИМЕР 1.1.8

Вычислить определитель

Решение. Разложим определитель по элементам третьей строки.

ПРИМЕР 1.1.9

Вычислить определитель

Решение. Прибавляя ко второй строке первую, умноженную на — 8,

получим Раскладывая этот определитель по элементам второй его строки, найдем

  • Определители n-го порядка с примерами
  • Курс математики

Сохранить или поделиться с друзьями


Вы находитесь тут:


На нашем сайте Вы можете получить решение задач и онлайн помощь

Подробнее

Стоимость мы сообщим в течение 5 минут
на указанный вами адрес электронной почты.
Если стоимость устроит вы сможете оформить заказ.

Скачать рамки A4


Скачать миллиметровки


Скачать шрифты ГОСТ

Сохранить или поделиться с друзьями

Заказать решение



Поиск математических формул

Определители третьего порядка

Высшая математика / Практикум по аналитической геометрии


Выражение вида

называется определителем третьего порядка.
Определитель третьего порядка имеет девять элементов, три строки, три столбца, две диагонали — главную и побочную.

Рис.1

Формула (1) показывает, что в раскрытом виде определитель содержит шесть членов. Для их определения существуют простые способы. Рассмотрим два из них.
Способ I. Выписываем все элементы определителя в том же порядке, как они расположены в определителе, и приписываем справа первые два столбца определителя:

Способ II. Возьмем со знаком плюс произведение элементов, стоящих на главной диагонали определителя, а также произведения элементов, стоящих на двух параллельных к ней линиях, содержащих по три элемента.
Произведения же элементов, стоящих на побочной диагонали и на двух параллельных к ней линиях, содержащих по три элемента, возьмем со знаком минус.
Алгебраическая сумма этих шести произведений дает значение определителя третьего порядка.

Пример. Вычислить определитель:

Решение.


Способ III. Способ разложения определителя по элементам какой-либо строки или столбца.
Если в определителе третьего порядка вычеркнуть одну строку и один столбец, на пересечении которых стоит некоторый элемент, то оставшиеся элементы образуют определитель второго порядка, который называется минором определителя Δ, соответствующим этому элементу. Например, минором определителя

соответствующим элементу будет определитель второго порядка

Чтобы вычислить определитель третьего порядка, нужно каждый элемент строки или столбца, по которым разлагается определитель, умножить на его минор, взятый со знаком плюс или минус в зависимости от того, будет ли сумма номеров зачеркнутых строки и столбца четным или нечетным числом. Например,

— разложение определителя Δ по первому столбцу.
Пример. Вычислить определитель

Решение.

Основные свойства определителей третьего порядка.
Свойство 1. Величина определителя не изменится, если за¬писать столбцы вместо строк, а строки вместо столбцов.
Свойство 2. При перестановке двух столбцов или строк определитель меняет знак.
Свойство 3. Чтобы умножить определитель на какое-нибудь число, достаточно умножить на это число все элементы какой-нибудь одной строки или какого-нибудь одного столбца.
Свойство 4. Если в определителе имеются две одинаковых строки или два одинаковых столбца, то он равен нулю.
Свойство 5. Величина определителя не изменится, если к элементам некоторого ряда прибавить элементы параллельного ряда, предварительно умножив их на постоянный множитель.

Тегивычисление определителей третьего порядкавычислить определитель третьего порядкавычислить определитель третьего порядка онлайнопределитель матрицы третьего порядкаопределитель третьего порядкаопределитель третьего порядка онлайнопределитель третьего порядка определениеопределитель третьего порядка формулаКалькулятор определителя

— примеры, Калькулятор определителя онлайн

Калькулятор определителя

— это онлайн-инструмент, который помогает найти определитель квадратной матрицы 3 x 3, используя формулу определителя. Найти определитель можно только квадратных матриц.

Что такое определительный калькулятор?

Калькулятор определителя помогает вычислить определитель квадратной матрицы 3 x 3. Когда мы находим сумму произведений элементов квадратной матрицы по какому-то заданному правилу, полученная таким образом величина называется определителем. Чтобы использовать Калькулятор определителя , введите значения в поля ввода.

Калькулятор определителя

ПРИМЕЧАНИЕ: Вводите значения не более 3 цифр.

Как пользоваться калькулятором определителя?

Выполните следующие действия, чтобы найти определитель матрицы 3 x 3 с помощью онлайн-калькулятора определителя.

  • Шаг 1: Перейдите к онлайн-калькулятору определителя Cuemath.
  • Шаг 2: Введите элементы матрицы в указанные поля ввода.
  • Шаг 3: Нажмите кнопку «Вычислить» , чтобы найти определитель.
  • Шаг 4: Нажмите кнопку «Сброс», чтобы очистить поля и ввести новые значения.

Как работает калькулятор определителя?

Определитель можно рассматривать как функцию, которая принимает элементы квадратной матрицы в качестве входных данных и выводит единственное значение. Детерминанты являются скалярными величинами. Определитель квадратной матрицы используется для нахождения обратной этой матрицы. Кроме того, нам потребуются определители, если мы решаем линейные уравнения, используя метод обращения матриц. Шаги для нахождения определителей матриц приведены ниже:

1. Матрица 2 x 2

Пусть \(A_{2\times 2}\) = \(\begin{bmatrix} a & b\\ c & d \end{bmatrix}\)

Формула для вычислить определитель следующим образом:

\(\begin{vmatrix}A_{2\times 2} \end{vmatrix}\) = (a x d) — (b x c)

2. 3 x 3 Матрица

Пусть \ (A_{3\times 3}\) = \(\begin{bmatrix} a & b & c\\ d& e& f\\ g& h & i \end{bmatrix}\)

Шаг 1: Умножаем номера привязок с соответствующей квадратной подматрицей.

\(\begin{vmatrix}A_{3\times 3} \end{vmatrix}\) = a . \(\begin{vmatrix} e & f\\ h & i \end{vmatrix}\) — b . \(\begin{vmatrix} d & f\\ g & i \end{vmatrix}\) + c . \(\begin{vmatrix} d & e\\ g & h \end{vmatrix}\)

Шаг 2: Используя формулу определителя матрицы 2 x 2, мы решаем выражение на шаге 1, как указано ниже.

\(\begin{vmatrix}A_{3\times 3} \end{vmatrix}\) = a(ei — fh) — b(di — fg) + c(dh — eg).

Хотите найти сложные математические решения за считанные секунды?

Воспользуйтесь нашим бесплатным онлайн-калькулятором, чтобы решить сложные вопросы. С Cuemath находите решения простыми и легкими шагами.

Записаться на бесплатный пробный урок

Решенные примеры с определителями

Пример 1: Найти определитель следующей матрицы \(\begin{bmatrix} 3 & 4 & 5\\ 1& 2& 3\\ 2& 0 & 9 \ end{bmatrix}\) и проверьте его с помощью онлайн-калькулятора определителей.

Решение:

Дано: \(A_{3\times 3}\) = \(\begin{bmatrix} 3 & 4 & 5\\ 1& 2& 3\\ 2& 0 & 9 \end{bmatrix}\ )

По формуле

\(A_{3\times 3}\) = \(\begin{bmatrix} a & b & c\\ d& e& f\\ g& h & i \end{bmatrix} \)

\(\begin{vmatrix}A_{3\times 3} \end{vmatrix}\) = a(ei — fh) — b(di — fg) + c(dh — eg)

Подставляя эти значений получаем

\(\begin{vmatrix}A_{3\times 3} \end{vmatrix}\) = 3 (2 x 9- 0 x 3) — 4 (1 x 9 — 0 x 3) + 5 (1 x 0 — 2 x 2)

\(\begin{vmatrix}A_{3\times 3} \end{vmatrix}\) = 22

Пример 2: Найдите определитель следующей матрицы \(\begin{bmatrix} -1 & 2 & -4\\ 7. 2& 6& 2.3\\ -5& 1.3 & 3 \end{bmatrix}\) и проверьте это с помощью онлайн-калькулятора определителя.

Решение:

Дано: \(A_{3\times 3}\) = \(\begin{bmatrix} -1 & 2 & -4\\ 7.2& 6& 2.3\\ -5& 1.3 & 3 \ конец{bmatrix}\)

По формуле

\(A_{3\times 3}\) = \(\begin{bmatrix} a & b & c\\ d& e& f\\ g& h & i \end{bmatrix}\ )

\(\begin{vmatrix}A_{3\times 3} \end{vmatrix}\) = a(ei — fh) — b(di — fg) + c(dh — eg)

Подстановка этих значений получаем

\(\begin{vmatrix}A_{3\times 3} \end{vmatrix}\) = -1 (6 x 3 — 2,3 x 1,3) — 2 (7,2 x 3 — 2,3 x (-5) ) — 4(7,2 x 1,3 — 6 x (-5))

\(\begin{vmatrix}A_{3\times 3} \end{vmatrix}\) = -238,65

Точно так же вы можете попробовать калькулятор определителей, чтобы найти определители для следующего:

  • \(\begin{bmatrix} 3.2 & -2 & 3\\ 1& 0& 4\\ -1& -7 & 4.7 \end{ бматрица}\)
  • \(\begin{bmatrix} 8.2 & -3 & 5\\ 1.9& 2.8& -3\\ 9& 1 & 1 \end{bmatrix}\)

☛ Математические калькуляторы:

Если все элементы на двух диагоналях определителя третьего порядка равны нулю, то каково значение определителя.

  • Курс
    • NCERT
      • Класс 12
      • Класс 11
      • Класс 10
      • Класс 9
      • Класс 8
      • 9
      • Класс 7 024
    • ИИТ ЕГЭ
  • Экзамен
    • JEE MAINS
    • JEE ADVANCED
    • X BOARDS
    • XII BOARDS
    • NEET
      • Neet Предыдущий год (по годам)
      • Физика Предыдущий год
      • Химия0024
      • Биология Предыдущий год
      • Нет Все образцы работ
      • Образцы работ по биологии
      • Образцы работ по физике
      • Образцы работ по химии
  • 0
  • Класс 12
  • Класс 11
  • Класс 10
  • Класс 9
  • Класс 8
  • Класс 7
  • Класс 6
    • Экзаменационный уголок
    • Онлайн-класс
    • Викторина
    • Задать вопрос в Whatsapp
    • Поиск Doubtnut
    • Английский словарь

    Теорема о подобных треугольниках: Признаки подобия треугольников. Средняя линия.

    Подобие треугольников и пропорциональные отрезки

    Теорема 1:

    Если на одной из сторон угла отметить равные между собой отрезки и через их концы провести параллельные прямые, то эти прямые отсекут на второй стороне также равные между собой отрезки.

     

    Доказательство:

    Докажем сначала лемму: Если в \(\triangle OBB_1\) через середину \(A\) стороны \(OB\) проведена прямая \(a\parallel BB_1\), то она пересечет сторону \(OB_1\) также в середине.


     

    Через точку \(B_1\) проведем \(l\parallel OB\). Пусть \(l\cap a=K\). Тогда \(ABB_1K\) — параллелограмм, следовательно, \(B_1K=AB=OA\) и \(\angle A_1KB_1=\angle ABB_1=\angle OAA_1\). Значит, по второму признаку \(\triangle OAA_1=\triangle B_1KA_1 \Rightarrow OA_1=A_1B_1\). Лемма доказана.


     

    Перейдем к доказательству теоремы. Пусть \(OA=AB=BC\), \(a\parallel b\parallel c\) и нужно доказать, что \(OA_1=A_1B_1=B_1C_1\).

     

    Таким образом, по данной лемме \(OA_1=A_1B_1\). Докажем, что \(A_1B_1=B_1C_1\). Проведем через точку \(B_1\) прямую \(d\parallel OC\), причем пусть \(d\cap a=D_1, d\cap c=D_2\). Тогда \(ABB_1D_1, BCD_2B_1\) — параллелограммы, следовательно, \(D_1B_1=AB=BC=B_1D_2\). Значит, по первому признаку \(\triangle A_1B_1D_1=\triangle C_1B_1D_2 \Rightarrow A_1B_1=B_1C_1\).

     

    Теорема Фалеса:

    Параллельные прямые отсекают на сторонах угла пропорциональные отрезки.


     

    Доказательство:

    Пусть параллельные прямые \(p\parallel q\parallel r\parallel s\) разбили одну из прямых на отрезки \(a, b, c, d\). Тогда вторую прямую эти прямые должны разбить на отрезки \(ka, kb, kc, kd\) соответственно.

     

    Проведем через точку \(A_1\) прямую \(p\parallel OD\) (\(ABB_2A_1\) — параллелограмм, следовательно, \(AB=A_1B_2\)). Тогда \(\triangle OAA_1 \sim \triangle A_1B_1B_2\) по двум углам. Следовательно, \(\dfrac{OA}{A_1B_2}=\dfrac{OA_1}{A_1B_1} \Rightarrow A_1B_1=kb\).

     

    Аналогично проведем через \(B_1\) прямую \(q\parallel OD \Rightarrow \triangle OBB_1\sim \triangle B_1C_1C_2 \Rightarrow B_1C_1=kc\) и т. д.

     

    Наиболее часто встречающиеся подобия треугольников:

     

    Теорема 2.

    Средняя линия треугольника отсекает от него подобный ему треугольник.


     

    Доказательство:

    Т.к. средняя линия — это отрезок, соединяющий середины двух сторон, то \(\dfrac{AB}{A_1B}=\dfrac{CB}{C_1B}=2\).

     

    Таким образом, по двум пропорциональным сторонам и углу между ними (\(\angle B\) — общий) \(\triangle A_1BC_1 \sim \triangle ABC\).

     

    Теорема 3.

    Треугольники, образованные диагоналями трапеции и основаниями, подобны.


     

    Доказательство:

    Т.к. \(AD\parallel BC \Rightarrow \angle OBC=\angle ODA\). \(\angle BOC=\angle AOD\) как вертикальные. Следовательно, по двум углам \(\triangle BOC\sim \triangle AOD\).

     

    Теорема 4.

    Высота прямоугольного треугольника, проведенная к гипотенузе, делит его на два подобных треугольника. \circ-\angle BA_1B_1=\angle BAB_1\).

     

    Таким образом, по двум углам (\(\angle O\) — общий) \(\triangle OAB\sim \triangle OA_1B_1\).

     

    Теорема 7.

    Если к окружности из одной точки проведены касательная и секущая, то:


     

    Доказательство:

    Т.к. угол между касательной и хордой, проведенной в точку касания, равен половине дуги, заключенной между ними, то \(\angle OKA=\frac12 \buildrel\smile\over{KA}=\angle KBA\).

     

    Следовательно, по двум углам (\(\angle O\) — общий) \(\triangle OKA\sim \triangle OKB\).

     

    Теорема 8.

    Если в окружности две хорды пересекаются, то:


     

    Доказательство:

    \(\angle A_1AB_1=\angle A_1BB_1\), т.к. опираются на одну и ту же дугу. \(\angle A_1CB=\angle B_1CA\), т.к. они вертикальные. Следовательно, по двум углам \(\triangle A_1BC\sim \triangle B_1C\).

     

    Аналогично \(\triangle ABC\sim \triangle A_1B_1C\).

     

    Подобные треугольники. Признаки подобия треугольников

    Содержание

    • Определение подобных треугольников
    • Коэффициент подобия треугольников
    • Перый признак подобия треугольников
    • Второй признак подобия треугольников
    • Третий признак подобия треугольников
    • Отношение площадей подобных треугольников

    Определение подобных треугольников

    Определение 1. Два треугольника называются подобными, если их углы соответственно равны и стороны одного треугольника пропорциональны сходственным сторонам другого треугольника.

    Определение 2. Сходственными называются стороны подобных треугольников, лежащих напротив равных углов.

    На рисунке 1 углы треугольников \( \small ABC \) и \( \small A_1B_1C_1 \) соответственно равны:

    Тогда стороны \( \small AB \) и \( \small A_1B_1 \), \( \small BC \) и \( \small B_1C_1 \), \( \small AC \) и \( \small A_1C_1 \) называются сходственными.

    Определение 1 можно понимать так: два треугольника подобны, если для них можно ввести обозначения и (Рис.1) так, что

    Если два треугольника и подобны, то это обозначают так:

    Коэффициент подобия треугольников

    Коэффициентом подобия треугольников k − это число, равное отношению сходственных сторон (см. формулу (2)).

    Перый признак подобия треугольников

    Теорема 1. Если два угла одного треугольника соответсвенно равны двум углам другого треугольника, то такие треугольники подобны.

    Доказательство. Пусть заданы два треугольника и и пусть , . Докажем, что (Рис.2).

    Поскольку сумма углов треугольника равна 180°, то можно записать:

    и, так как , , получим:

    Таким образом углы треугольника соответственно равны углам треугольника . Покажем, теперь, что стороны одного треугольника пропорциональны сходственным сторонам другого треугольника, т. е. выполнено равенство (2).

    Площади треугольников и по двум сторонам и углу между ними можно вычислить формулами:

    Из (3) и (4), и из следует:

    С другой стороны:

    Из (6) и (7), и из следует:

    Левые части уравнения (5) и (8) равны. Следовательно равны и правые части:

    Умножая левую и правую части уравнения (9) на , получим:

    Продолжая аналогичные рассуждения, получим:

    Сравнивая (8) и (11), получим:

    Умножая левую и правую части уравнения (12) на , получим:

    Из (10) и (13), получим:

    То есть стороны треугольника пропорциональны сходственным сторонам треугольника . Что и требовалось доказать.

    Второй признак подобия треугольников

    Теорема 2. Если две стороны одного треугольника пропорциональны двум сторонам другого треугольника и углы между этими сторонами равны, то такие треугольники подобны.

    Доказательство. Пусть заданы два треугольника и и пусть , . Докажем, что (Рис.3).

    Рассмотрим треугольник у которого

    Из условия (15) следует, что треугольники и подобны (по первому признаку подобия треугольников). Следовательно:

    Но по условию теоремы . Поэтому . Треугольники и равны по двум сторонам и углу между ними (сторона AB общая, , (поскольку и )). Следовательно и поскольку , то .

    Получили, что и . Тогда по первому признаку подобия треугольников .

    Третий признак подобия треугольников

    Теорема 3. Если три стороны одного треугольника пропорциональны трем сторонам другого треугольника, то такие треугольники подобны.

    Доказательство. Пусть стороны треугольников пропорциональны:

    Докажем, что . Рассотрим треугольник у которого , (Рис.3). Треугольники и подобны по первому признаку подобия треугольников. Тогда выполнено следующее равенство:

    Сравнивая равенства (16) и (17) получаем: , .

    Из этих рассуждений следует, что треугольники и равны по трем сторонам (см. статью Треугольники. Признаки равенства треугольников). Тогда , а поскольку , то . Следовательно, по второму признаку подобия треугольников, треугольники и подобны: .

    Отношение площадей подобных треугольников

    Теорема 4. Отношение площадей подобных треугольников равно квадрату коэффициента подобия.

    Доказательство. Пусть треугольники и подобны. Тогда

    и

    где -коэффициент подобия.

    Площади треугольников и по двум сторонам и углу между ними равны:

    Тогда

    Теоремы о подобных треугольниках

    1. Теорема о делителях сторон

    Если треугольник ADE и BC параллельны DE, то AB BD = AC CE

    Чтобы показать, что это так, проведите прямую BF параллельно AE, чтобы составить параллелограмм BCEF:

    Треугольники ABC и BDF имеют одинаковые углы и поэтому подобны (Почему? См. раздел под названием AA на странице Как Чтобы определить, подобны ли треугольники. )

    • Сторона AB соответствует стороне BD, а сторона AC соответствует стороне BF.
    • Итак, AB/BD = AC/BF
    • Но BF = CE
    • Итак, AB/BD = AC/CE

    Теорема о биссектрисе угла

    Если треугольник ABC делит (делит пополам) угол BAC, то AB BD = AC DC

    Чтобы показать, что это верно, мы можем обозначить треугольник следующим образом: 9000 5

    • Угол BAD = Угол DAC = x°
    • Угол ADB = y°
    • Угол ADC = (180−y)°

    По закону синусов в треугольнике ABD: sin(x) BD = sin(y) AB

    Умножить обе стороны на AB: sin(x)AB БД = sin(y) 1

    Разделите обе части на sin(x): AB BD = sin(y) sin(x) 90 005

     

    По закону синусов в треугольник ACD: sin(x) DC = sin(180−y) AC

    Умножить обе стороны на AC: sin(x)AC DC = sin(1 80−у) 1

    Разделите обе части на sin(x): AC DC = sin(180−y) sin(x)

    Но sin(180−y) = sin(y) ) : AC DC = sin(y) sin(x)

    Оба AB BD и AC DC равны sin(y) sin(x) , поэтому:

    AB BD = AC 90 008 DC

    В частности, если треугольник ABC равнобедренный, то треугольники ABD и ACD равны:

    И тот же результат верен:

    AB BD = AC DC 900 05

    3.

    Площадь и подобие

    Если два подобных треугольника имеют сторон в отношении x:y,

    , то их площади относятся друг к другу x 2 :y 2

    Пример:

    Эти два треугольника подобны, стороны которых относятся как 2:1 (сторона одного вдвое длиннее другого) :

    Что можно сказать об их районах?

    Ответ прост, если мы просто нарисуем еще три линии:

    Мы можем видеть, что маленький треугольник вписывается в большой треугольник четыре раза .

    Итак, когда длины равны в два раза больше , площадь в четыре раза больше

    Таким образом, отношение их площадей равно 4:1

    Мы также можем записать 4:1 как 2 2 :1

    Общий случай:

    Треугольники ABC и PQR подобны и имеют отношение сторон x:y

    Площади можно найти по этой формуле из площади треугольника:

    Площадь ABC = 1 2 грех (A)

    Площадь PQR = 1 2 qr sin(P)

    И мы знаем, что длины треугольников находятся в соотношении x:y

    q/b = y/x, поэтому: q = by/x

    и r /c = y/x, поэтому r = cy/x

    Кроме того, поскольку треугольники подобны, углы A и P одинаковы:

    A = P

    Теперь мы можем сделать некоторые вычисления :

    Площадь треугольника PQR : 1 2 qr sin(P)

    Введите «q = by/x», «r = cy/x» и «P=A»: 1 2 (by)(cy) sin(A) (x)(x)

    Упрощение: 1 2 до н. э. 2 sin(A) x 2

    Задний диапазон: г 2 x 2 × 1 2 bc sin(A)

    Что равно: y 2 x 2 × Площадь треугольника ABC

    Таким образом, мы получим это соотношение:

    Площадь треугольника ABC: Площадь треугольника PQR = x 2 : Y 2

    Каковы теорема сходства треугольника?

    Обновлено 14 мая 2018 г.

    Автор Bert Markgraf

    Подобные треугольники имеют одинаковую форму, но не обязательно одинаковый размер. Когда треугольники подобны, они имеют много одинаковых свойств и характеристик. Теоремы подобия треугольников определяют условия, при которых два треугольника подобны, и они имеют дело со сторонами и углами каждого треугольника. Как только конкретная комбинация углов и сторон удовлетворяет теоремам, вы можете считать треугольники подобными.

    TL;DR (слишком длинный; не читал)

    Существуют три теоремы о подобия треугольников, которые определяют, при каких условиях треугольники подобны:

    • Если два угла одинаковы, то и третий угол тоже треугольники подобны.
    • Если три стороны находятся в одинаковых пропорциях, треугольники подобны.
    • Если две стороны имеют одинаковые пропорции и угол между ними одинаков, треугольники подобны.

    Теоремы AA, AAA и угол-угол

    Если два угла двух треугольников одинаковы, треугольники подобны. Это становится ясным из наблюдения, что сумма трех углов треугольника должна составлять 180 градусов. Если два угла известны, третий можно найти, вычитая два известных угла из 180. Если три угла двух треугольников одинаковы, треугольники имеют одинаковую форму и подобны.

    Теорема SSS или Side-Side-Side

    Если все три стороны двух треугольников одинаковы, треугольники не только подобны, они конгруэнтны или идентичны. Для подобных треугольников три стороны двух треугольников должны быть пропорциональны. Например, если один треугольник имеет стороны 3, 5 и 6 дюймов, а второй треугольник имеет стороны 9, 15 и 18 дюймов, каждая из сторон большего треугольника в три раза длиннее одной из сторон меньшего треугольника. Стороны пропорциональны друг другу, а треугольники подобны.

    Теорема SAS или сторона-угол-сторона

    Два треугольника подобны, если две стороны двух треугольников пропорциональны и угол между ними одинаков. Например, если две стороны треугольника равны 2 и 3 дюймам, а стороны другого треугольника равны 4 и 6 дюймам, стороны пропорциональны, но треугольники могут быть не подобны, поскольку две третьи стороны могут быть любой длины. Если углы между ними одинаковы, то все три стороны треугольников пропорциональны и треугольники подобны.

    Другие возможные комбинации углов и сторон

    Если для двух треугольников выполняется одна из трех теорем о подобии треугольников, треугольники подобны. Но есть и другие возможные комбинации боковых углов, которые могут гарантировать или не гарантировать сходство.

    Для конфигураций, известных как угол-угол-сторона (AAS), угол-бок-угол (ASA) или сторона-угол-угол (SAA), не имеет значения, насколько велики стороны; треугольники всегда будут подобны. Эти конфигурации сводятся к теореме AA «угол-угол», которая означает, что все три угла одинаковы, а треугольники подобны.

    Однако конфигурации бок-бок-бок или угол-бок-бок не обеспечивают сходства. (Не путайте сторона-бок-угол с стороной-угол-сторона; «стороны» и «углы» в каждом имени относятся к порядку, в котором вы встречаете стороны и углы.) В некоторых случаях, например, для прямого -угольные треугольники, если две стороны пропорциональны и углы, которые не входят в них, равны, треугольники подобны. Во всех остальных случаях треугольники могут быть, а могут и не быть.

    Подобные треугольники вписываются друг в друга, могут иметь параллельные стороны и масштабироваться от одного к другому.

    Делимость суммы и произведения: Делимость произведения, суммы и разности чисел — урок. Математика, 5 класс.

    Конспект урока по теме «Делимость суммы и произведения»

    Конспект урока по математике 5 класса по теме «Делимость суммы и произведения»

    УМК Математика. Арифметика. Геометрия 5 класс. Авторы Е. А. Бунимович, Г. В. Дорофеев и др.

    Предмет: математика
    Класс: 5
    Учитель:
    Белокобыленко Иван Алексеевич
    Тема урока: делимость суммы и произведения.
    Тип урока: урок изучения нового материала.

    Цель урока: узнать правило делимости суммы чисел и правило делимости произведения чисел.

    Задачи урока:

    1.     Образовательные

    • сформировать вычислительные навыки;

    • знать правила делимости суммы и произведения.

    2.     Развивающие

    • развивать познавательную активность;

    • развивать математическую речь;

    • развивать умение анализировать, сравнивать, строить аналогии;

    • развивать логическое мышление, память, внимание;

    • прививать интерес к предмету.

    3.     Воспитательные

    • учить прислушиваться к мнению своих товарищей;

    • уметь слушать и вступать в диалог;

    • формировать внимательность и аккуратность в вычислениях;

    • развивать навыки контроля и самоконтроля, самооценки.

    Оборудование: компьютер, проектор, экран, доска, презентация.

    Ход урока

    I. Организационный этап.

    Учитель: здравствуйте, ребята, садитесь. Проверьте все ли вы приготовили для урока (учебники, тетради, дневники, карандаши, ручки, черновики). Ну что же, давайте начнём!

    Учащиеся: включаются в работу и в ход урока.

    II. Актуализация опорных знаний.

    Учитель: прежде чем мы продолжим нашу работу, давайте вспомним материал, который изучили на прошлом уроке.

    • Повторение ранее изученного материала: мини-опрос по понятиям, устный счет по цепочке.

    III. Вхождение в тему урока, создание условий для изучения нового материала.

    Учитель: Ребята! Сегодня мы с вами познакомимся с новым материалом как делимость суммы и произведения.

    Учащиеся: записывают тему в тетрадь.

    IV. Изучение нового материала.

    Учитель: рассмотрим произведение чисел 214 и 33. Возникает вопрос: как нам понять делится ли произведение этих чисел на 11? Давайте преобразуем это произведение и посмотрим: 214 * 33 = 214 * (11 * 3) = 11 * (214 * 33) – переместительное свойство умножения. Теперь мы видим, что данное выражение делится на 11, следовательно и произведение этих чисел делится на 11.

    Ввести правило делимости произведения, рассмотреть еще один пример 32 * 99. Делится ли произведение на 9?

    Правило делимости суммы: 96 = 40 + 24 + 32, каждое слагаемое делится на 4, значит и 96 делится на четыре.

    V. Физкультминутка.

    1) Гимнастика для глаз;

    2) Гимнастика для шеи, кистей.

    VI. Закрепление изученного материала.

    Решение заданий №350

    351

    352 (а)

    356

    358 (а, б)

    VII. Объяснение домашнего задания.

    Домашнее задание: С. 102 – 103, №352 (б), 358 (в, г), 360 (а, б)

    VIII. Рефлексия. Подведение итогов урока.

    Учитель: Ребята, спасибо за активную работу. Скажите мне, пожалуйста, как вы считаете сложно ли вам получалось освоить тему нашего урока? (ответ нескольких учащихся).

    Отлично, ну что наш урок подошел к концу. Всем спасибо, до свидания!

    Урок 22. Делимость произведения — гдз по математике для 6 класса Зубарева И.И., Мордкович А.Г.

    Класс

    • 1 класс

    • 2 класс

      • Английский язык
      • Математика
    • 3 класс

      • Русский язык
      • Английский язык
      • Математика
    • 4 класс

      • Русский язык
      • Английский язык
      • Математика
    • 5 класс

      • Русский язык
      • Английский язык
      • Математика
      • Биология
    • 6 класс

      • Русский язык
      • Английский язык
      • Математика
      • Биология
    • 7 класс

      • Русский язык
      • Английский язык
      • Математика
      • Биология
      • Физика
      • Химия
    • 8 класс

      • Русский язык
      • Английский язык
      • Математика
      • Биология
      • Физика
      • Химия
    • 9 класс

      • Русский язык
      • Английский язык
      • Математика
      • Биология
      • Физика
      • Химия
    • 10 класс

      • Английский язык
      • Биология
      • Физика
      • Химия
    • 11 класс

      • Английский язык
      • Биология
      • Химия

    6 КЛАСС

    Урок 22.
    Делимость произведения
    Ответ: да, можно по 318 книг.
    Ответ: да, можно по 15 роз в букете
    Ответ: да, по 15 конфет
    Свойства многочленов:
    1) Члены многочлена можно менять местами.
    2) Прибавление к многочлену нуля или нулевого многочлена не изменят его.
    3) В многочлене можно приводить подобные члены.
    
    Чтобы раскрыть скобки, перед которыми стоит знак "+", скобки можно опустить, не меняя знаки слагаемых в скобках.
    Чтобы раскрыть скобки, перед которыми стоит знак "−", скобки можно опустить, изменив знак каждого слагаемого в скобках, не противоположный.
    Чтобы заключить многочлен в скобки со знаком "+" перед ними, нужно записать в скобках все его члены с теми же знаками.
    Чтобы заключить многочлен в скобки со знаком "−" перед ними, нужно записать в скобках все его члены с противоположными знаками.
    
    Алгебраическая дробь не определена, когда при подстановке числового значения вместо переменной получается деление на нуль.
    
    Для разложения многочлена на множители можно использовать:
    1) вынесение общего множителя за скобки;
    2) применение формул сокращенного умножения;
    3) способ группировки;
    4) выделение полного квадрата;
    5) применение разных способов разложения на множители. 
    
    Внимательно читайте условие задания.
    
    Соседи числа — это число, которое предшествует этому числу при счете (предыдущее число), и число, которое при счёте следует за ним.
    
    Увеличить число на несколько единиц - использовать действие сложение, знак"+" Уменьшить на несколько единиц- использовать действие вычитание, знак "-".
    
    Сумма, разность, произведение рациональных чисел является рациональным числом (без деления на нуль).
    
    Сложение — это математическое действие. Числа, которые складываются, называются слагаемыми. 
    
    Квадрат разности двух чисел равен квадрату первого числа минус удвоенное произведение первого и второго чисел плюс квадрат второго числа.
    
    Непростые натуральные числа, больше 1, называют составными числами.
    
    Вычитание — обратное сложению арифметическое действие, посредством которого от одной величины отнимается другая величина.
    
    Куб суммы двух чисел равен кубу первого числа плюс утроенное произведение квадрата первого числа и второго плюс утроенное произведение первого числа и квадрата второго плюс куб второго числа. 
    
    Чтобы определить, делиться ли одно натуральное число на другое, можно это делимое число разложить на множители.
    
    Задача на нахождение суммы всегда решается действием сложения.
    Задача на нахождение остатка решается действием вычитания. знак "-"
    
    Разность квадратов двух чисел равно произведению суммы этих чисел и их разности.
    
    Ответ: 36 яблок и 48 яблок.
    
    Ответ: 212 пачек и 28 пачек
    
    Ответ: n/7 конфет
    Разность кубов двух чисел равна произведению разности этих чисел на неполный квадрат их суммы.
    
    Ответ: 4
    Натуральные числа не могут быть отрицательными. Поэтому натуральное число получаем, если мы из большего отнимаем меньшее, но не наоборот.
    
    Ответ: 60
    

    Вопросники:

    Пропуски:

    Выполните все задания.

    ВерноНе верноИногда верноЗависит от условий задачи

    код гольф — Числа, которые делятся на сумму и произведение своих цифр

     p
    \Ай
    \&
    >(&]&|0
      <*&d
     &~bN
      10
     ( )/+
     /*
     

    Попробуйте онлайн!

    Объяснение

    Это самая сложная (и самая длинная) программа, которую я когда-либо писал на Jellyfish. Я понятия не имею, смогу ли я объяснить это понятным образом, но, думаю, мне придется попытаться.

    Jellyfish предоставляет довольно общий оператор итерации, \ , что очень помогает при "поиске N-го чего-то ". Одна из его семантик — «итерировать функцию по значению до тех пор, пока отдельная тестовая функция не даст что-то правдивое» (на самом деле тестовая функция получает и текущий, и последний элемент, но мы заставим ее смотреть только на текущий элемент) . Мы можем использовать это для реализации функции «следующий допустимый номер». Другая перегрузка \ — «итерация функции по начальному значению N раз». Мы можем использовать нашу предыдущую функцию и повторить ее на 0 N раз, где N — ввод. Все это настроено довольно лаконично в этой части кода:

     p
    \Ай
    \&
    > 0
     

    (Причины, по которым 0 , фактический ввод в результирующую функцию, закончились, немного сложны, и я не буду вдаваться в них здесь. )

    Проблема со всем этим в том, что мы победим. не передавать текущее значение тестовой функции вручную. За нас это сделает оператор \. Итак, теперь у нас есть единая унарная функция (с помощью композиций, хуков, вилок и каррирования), которая принимает число и сообщает нам, является ли оно допустимым числом (то есть тем, которое делится на сумму цифр и произведение цифр). Это довольно нетривиально, когда вы не можете ссылаться на аргумент. Всегда. Вот эта красота:

     (&]&|
      <*&d
     &~bN
      10
     ( )/+
     /*
     

    ( является унарным хуком , что означает, что он вызывает функцию ниже ( f ) на своем входе (текущее значение x ), а затем передает их обе в тестовую функцию в right ( g ), то есть вычисляет g(f(x), x) .

    В нашем случае f(x) является еще одной составной функцией, которая получает пару с произведением цифр и суммой цифр из х . Это означает, что g будет функцией, имеющей все три значения для проверки правильности x .

    Начнем с рассмотрения того, как f вычисляет сумму цифр и произведение цифр. Это f :

     &~b
      10
     ( )/*
     /+
     

    и тоже композиция (но наоборот). ~ является каррированием, поэтому 10~b дает функцию, которая вычисляет десятичные цифры числа, и поскольку мы передаем это в и справа, это первое, что произойдет с вводом x . Остальные используют этот список цифр для вычисления их суммы и произведения.

    Чтобы вычислить сумму, мы можем сложить с ней , что равно /+ . Точно так же, чтобы вычислить произведение, мы умножаем его на /* . Чтобы объединить оба этих результата в пару, мы используем пару хуков ( и ) . Структура этого:

     ()г
    ф
     

    (Где f и g — произведение и сумма соответственно.) Попробуем разобраться, почему это дает нам пару f(x) и g(x) . Обратите внимание, что правый хук ) имеет только один аргумент. В этом случае подразумевается, что другой аргумент равен ; , который заключает свои аргументы в пару. Кроме того, хуки также могут использоваться как бинарные функции (что и будет здесь), и в этом случае они просто применяют внутреннюю функцию только к одному аргументу. Так что действительно ) для одной функции g дает функцию, которая вычисляет [x, g(y)] . Используя это в левом хуке вместе с f , мы получаем [f(x), g(y)] . Это, в свою очередь, используется в унарном контексте, что означает, что на самом деле он вызывается с x == y , и поэтому мы получаем [f(x), g(x)] , как требуется. Фу.

    Остается только одно: наша предыдущая тестовая функция g . Напомним, что он будет называться g([p, s], x) , где x — это текущее входное значение, p — его произведение цифр, а s — сумма цифр. Это г :

     &]&|
      <*&d
        Н
     

    Чтобы проверить делимость, мы, очевидно, будем использовать модуль, который равен | в Медузах. Несколько необычно то, что он берет свой правый операнд по модулю своего левого операнда, что означает, что аргументы g уже находятся в правильном порядке (такие арифметические функции, как эта, автоматически перебирают списки, так что это будет вычислять два отдельных модуля). бесплатно). Наше число делится и на произведение, и на сумму, если в результате получается пара нулей. Чтобы проверить, так ли это, мы рассматриваем пару как список цифр с основанием 2 ( д ). Результат этого равен нулю, только когда оба элемента пары равны нулю, поэтому мы можем инвертировать результат этого ( N ), чтобы получить истинное значение того, делят ли входные данные оба значения. Обратите внимание, что | , d и N просто составлены вместе с парой и s.

    К сожалению, это еще не все. Что, если цифровое произведение равно нулю? Деление и по модулю на ноль возвращают ноль в Jellyfish. Хотя это может показаться несколько странным соглашением, на самом деле оно оказывается несколько полезным (поскольку нам не нужно проверять наличие нуля перед выполнением по модулю). Однако это также означает, что мы можем получить ложное срабатывание, если сумма цифр действительно делит ввод, но произведение цифр равно нулю (например, ввод 10 ).

    Мы можем исправить это, умножив наш результат делимости на числовое произведение (поэтому, если числовое произведение равно нулю, оно также превратит наше истинное значение в ноль). Оказывается, проще умножить результат делимости на пару произведение и сумма, а затем извлечь результат из произведения.

    Чтобы умножить результат на пару, нам нужно вернуться к более раннему значению (паре). Делается это вилкой ( ] ). Вилки похожи на крючки на стероидах. Если дать им две функции f и g , они представляют бинарную функцию, которая вычисляет f(a, g(a, b)) . В нашем случае a — это пара произведение/сумма, b — текущее входное значение, g — наш тест на делимость, а f — умножение. Итак, все это вычисляет [p, s] * ([p, s] % x == [0, 0]) .

    Теперь осталось только извлечь первое значение this, которое является окончательным значением тестовой функции, используемой в итераторе. Это так же просто, как сочинить ( & ) вилка с головкой функция < , которая возвращает первое значение списка.

    Произведение, кратное сумме квадратов

    Произведение, кратное сумме квадратов
    ПРЕДЛОЖЕНИЕ: Произведение n различных простых чисел не может быть
                  делится на сумму своих квадратов, если n < 5.
    Доказательство этого предложения для случаев n = 1, 2, 3 и 4
    предлагает интересную последовательность постепенно усложняющихся
    демонстрации.  2 + 4
    p' < ------------------
    q ( ​​г - SR (3))
    Оценка этого неравенства почленно дает
    д р р 4
    p' < ------ + --- ------- + ----------
    р-SR(3) q r-SR(3) q(r-SR(3))
    Поскольку 2 < r < q, второе слагаемое должно быть меньше 2,366.., а
    третий член меньше 0,788... Поэтому имеем
    д
    p' < ------- + 3,154...
    р-СР(3)
    что показывает, что p' < q для всех q > 15. Проверка случаев с
    2 < r < q < 15 показывает, что в этом диапазоне нет решений, поэтому p' должно
    быть меньше q. Отсюда следует, что p',q,r является другим решением в
    различные положительные целые числа с меньшим наибольшим членом, чем p,
    противоречащее предположению.
    Это завершает доказательство того, что не существует четырех (или меньше) простых чисел.
    произведение которых делится на сумму их квадратов. Однако,
    существует много наборов из пяти простых чисел с этим свойством.
    наименьший пример [3,5,11,192
    Для любого такого набора простых чисел мы ссылаемся на отношение наибольшего к
    наименьшее, как «соотношение сторон», поэтому соотношение сторон выше
    например 23/3 = 7,666.
            

    0 3 в кубе: Таблица кубов

    Таблица кубов

    К содержанию

    Куб числа — есть данное число, возведенное в третью степень. «Кубом» оно называется, потому что такая операция используется для нахождения объема куба (по аналогии с квадратом числа). То есть, чтобы найти объем куба, необходимо возвести в третью степень длину ребра куба. Точно также, чтобы найти куб числа нужно возвести его в третью степень. В таблице приведены значения кубов натуральных чисел от 1 до 100.

    1 3 = 1
    2 3 = 8
    3 3 = 27
    4 3 = 64
    5 3 = 125
    6 3 = 216
    7 3 = 343
    8 3 = 512
    9 3 = 729
    10 3 = 1000
    11 3 = 1331
    12 3 = 1728
    13 3 = 2197
    14 3 = 2744
    15 3 = 3375
    16 3 = 4096
    17 3 = 4913
    18 3 = 5832
    19 3 = 6859
    20 3 = 8000
    21 3 = 9261
    22 3 = 10648
    23 3 = 12167
    24 3 = 13824
    25 3 = 15625
    26 3 = 17576
    27 3 = 19683
    28 3 = 21952
    29 3 = 24389
    30 3 = 27000
    31 3 = 29791
    32 3 = 32768
    33 3 = 35937
    34 3 = 39304
    35 3 = 42875
    36 3 = 46656
    37 3 = 50653
    38 3 = 54872
    39 3 = 59319
    40 3 = 64000
    41 3 = 68921
    42 3 = 74088
    43 3 = 79507
    44 3 = 85184
    45 3 = 91125
    46 3 = 97336
    47 3 = 103823
    48 3 = 110592
    49 3 = 117649
    50 3 = 125000
    51 3 = 132651
    52 3 = 140608
    53 3 = 148877
    54 3 = 157464
    55 3 = 166375
    56 3 = 175616
    57 3 = 185193
    58 3 = 195112
    59 3 = 205379
    60 3 = 216000
    61 3 = 226981
    62 3 = 238328
    63 3 = 262144
    64 3 = 262144
    65 3 = 274625
    66 3 = 287496
    67 3 = 300763
    68 3 = 314432
    69 3 = 328509
    70 3 = 343000
    71 3 = 357911
    72 3 = 373248
    73 3 = 389017
    74 3 = 405224
    75 3 = 421875
    76 3 = 438976
    77 3 = 456533
    78 3 = 474552
    79 3 = 493038
    80 3 = 512000
    81 3 = 531441
    82 3 = 551368
    83 3 = 571787
    84 3 = 592704
    85 3 = 614125
    86 3 = 636056
    87 3 = 658503
    88 3 = 681472
    89 3 = 704969
    90 3 = 729000
    91 3 = 753571
    92 3 = 778688
    93 3 = 804357
    94 3 = 830584
    95 3 = 857375
    96 3 = 884736
    97 3 = 912673
    98 3 = 941192
    99 3 = 970299
    100 3 = 1000000

    Другие заметки по алгебре и геометрии

    Полезная информация?

    Таблица кубов

    Таблица кубов

    Определение Калькулятор — куб числа Таблица кубов

    Скачать таблицу кубов

    Определение. Куб числа — есть данное число, возведенное в третью степень.

    a3 = a · a · a

    «Кубом» оно называется, потому что такая операция аналогична вычислению объема куба.

    Калькулятор для вычисления куба числа

    3 = 827 ≈ 0.2962962962962963

    Ниже приведены две удобные таблицы кубов натуральных чисел от 1 до 100.


    Таблица кубов чисел от 1 до 100

    13 = 1

    23 = 8

    33 = 27

    43 = 64

    53 = 125

    63 = 216

    73 = 343

    83 = 512

    93 = 729

    103 = 1000

    113 = 1331

    123 = 1728

    133 = 2197

    143 = 2744

    153 = 3375

    163 = 4096

    173 = 4913

    183 = 5832

    193 = 6859

    203 = 8000

    213 = 9261

    223 = 10648

    233 = 12167

    243 = 13824

    253 = 15625

    263 = 17576

    273 = 19683

    283 = 21952

    293 = 24389

    303 = 27000

    313 = 29791

    323 = 32768

    333 = 35937

    343 = 39304

    353 = 42875

    363 = 46656

    373 = 50653

    383 = 54872

    393 = 59319

    403 = 64000

    413 = 68921

    423 = 74088

    433 = 79507

    443 = 85184

    453 = 91125

    463 = 97336

    473 = 103823

    483 = 110592

    493 = 117649

    503 = 125000

    513 = 132651

    523 = 140608

    533 = 148877

    543 = 157464

    553 = 166375

    563 = 175616

    573 = 185193

    583 = 195112

    593 = 205379

    603 = 216000

    613 = 226981

    623 = 238328

    633 = 250047

    643 = 262144

    653 = 274625

    663 = 287496

    673 = 300763

    683 = 314432

    693 = 328509

    703 = 343000

    713 = 357911

    723 = 373248

    733 = 389017

    743 = 405224

    753 = 421875

    763 = 438976

    773 = 456533

    783 = 474552

    793 = 493039

    803 = 512000

    813 = 531441

    823 = 551368

    833 = 571787

    843 = 592704

    853 = 614125

    863 = 636056

    873 = 658503

    883 = 681472

    893 = 704969

    903 = 729000

    913 = 753571

    923 = 778688

    933 = 804357

    943 = 830584

    953 = 857375

    963 = 884736

    973 = 912673

    983 = 941192

    993 = 970299

    1003 = 1000000

     Распечатать таблицу кубов

    Таблица кубов

    0123456789
    00182764125216343512729
    11000133117282197274433754096491358326859
    2800092611064812167138241562517576196832195224389
    327000297913276835937393044287546656506535487259319
    464000689217408879507851849112597336103823110592117649
    5125000132651140608148877157464166375175616185193195112205379
    6216000226981238328250047262144274625287496300763314432328509
    7343000357911373248389017405224421875438976456533474552493039
    8512000531441551368571787592704614125636056658503681472704969
    9729000753571778688804357830584857375884736912673941192970299

     Распечатать таблицу кубов

    © 2011-2023 Довжик Михаил
    Копирование материалов запрещено.

    Добро пожаловать на OnlineMSchool.
    Меня зовут Довжик Михаил Викторович. Я владелец и автор этого сайта, мною написан весь теоретический материал, а также разработаны онлайн упражнения и калькуляторы, которыми Вы можете воспользоваться для изучения математики.

    Если Вы хотите связаться со мной, имеете вопросы, предложения или хотите помочь развивать сайт OnlineMSchool пишите мне [email protected]

    3 = 4 х 4 х 4 = 64 \]

    Поделитесь этой ссылкой для ответа: help
    Вставьте эту ссылку в электронное письмо, текст или социальные сети.


    Получить виджет для этого калькулятора

    © Calculator Soup

    Поделитесь этим калькулятором и страницей

    Калькулятор Использование

    Найдите значение числа в кубе n . Введите положительные или отрицательные целые числа, десятичные числа или научную нотацию E.

    Что такое число в кубе?

    Любое число n с показателем степени 3 записывается как . Вы произносите это как « n в кубе» или « n в третьей степени». Чтобы получить куб числа, умножьте его само на себя 3 раза.

    Следовательно, формула куба n³ = n × n × n .

    Что такое идеальный куб?

    Идеальный куб получается при возведении в куб целого числа или целого числа без десятичных знаков и дробей. Например, 3 в кубе записывается как 3³, а 3³ = 3 × 3 × 3 = 27,9.0003

    Поскольку 3 — целое число, 27 — совершенный куб.

    Представьте себе блок меньших кубиков 3 в высоту, 3 в ширину и 3 в глубину, напоминающий кубик Рубика. По сути, это 3 набора по 9 блоков, расположенных по схеме 3 х 3. Поскольку 3 x 3 равно 9, что также равно 3 в квадрате или 3 2 , вам просто нужно снова умножить на 3, чтобы получить 3 в кубе, 3 3 = 27.

    Это может помочь представить любое число в кубе как набор блоков. 10 3 или 10 в кубе, например, будет набором блоков 10 в высоту, 10 в ширину и 10 в глубину. Одна грань куба будет иметь набор 10 93 = 1000 \]

    Изображение ниже представляет собой большой куб из 10³ = 1000 меньших кубов. Обратите внимание, что каждый слой равен 10 х 10 = 100 кубов.

    Числа от 0 до 10 в кубе и полученные в результате совершенные кубы

    • 0 в кубе равно 0³ = 0 × 0 × 0 = 0
    • 1 куб равен 1³ = 1 × 1 × 1 = 1
    • 2 в кубе равно 2³ = 2 × 2 × 2 = 8
    • 3 в кубе равно 3³ = 3 × 3 × 3 = 27
    • 4 в кубе равно 4³ = 4 × 4 × 4 = 64
    • 5 в кубе равно 5³ = 5 × 5 × 5 = 125
    • 6 в кубе равно 6³ = 6 × 6 × 6 = 216
    • 7 в кубе равно 7³ = 7 × 7 × 7 = 343
    • 8 в кубе равно 8³ = 8 × 8 × 8 = 512
    • 9 в кубе равно 9³ = 9 × 9 × 9 = 729
    • 10 в кубе равно 10³ = 10 × 10 × 10 = 1000

    См. наш список первых 100 идеальных кубов.

    Куб отрицательных чисел

    При возведении в куб отрицательного числа результатом всегда будет отрицательное число. Но важно понимать, как использование круглых скобок влияет на интерпретацию операции.

    Знак «минус» перед числом или вне круглых скобок означает «возьмите отрицательное число в кубе», как в

    • -2³ означает -(2 × 2 × 2) = -8
    • -(2)³ означает -(2 × 2 × 2) = -8

    Знак «минус» внутри скобок означает «возвести отрицательное число в куб», как в

    • (-2)³ означает (-2 × -2 × -2) = -8

    Как видите, вы всегда получите отрицательный результат с нечетным показателем степени и отрицательным знаком, независимо от того, находится ли он внутри или снаружи круглых скобок. Но когда у вас четный показатель степени, результат зависит от интерпретации отрицательного знака и наличия скобок.

    Может быть полезно использовать круглые скобки, чтобы четко указать, какое вычисление степени вы собираетесь вычислять. Например, при четных показателях и отрицательном знаке круглые скобки влияют на результат.

    • -4² означает -(4 × 4) = -16
    • -(4)² означает -(4 × 4) = -16
    • (-4)² означает (-4 × -4) = 16

    Вы можете видеть, что Square Calculator использует эту строгую интерпретацию. Без круглых скобок -4² означает «минус 4²», что равно -16.

    Дополнительная литература

    Википедия «Куб (алгебра)» на https://en.wikipedia.org/wiki/Куб_(алгебра)

     

    Подписаться на калькуляторSoup:

    Куб от 1 до 100 | Значения кубиков от 1 до 100

    Кубики от 1 до 100 — это список кубиков всех чисел от 1 до 100. Значения кубиков от 1 до 100 варьируются от 1 до 1000000. Запоминание этих значений поможет учащимся упростить трудоемкие уравнения быстро. Куб от 1 до 100 в экспоненциальной форме выражается как (x) 3 .

    Изучение кубов от 1 до 100 может помочь учащимся распознавать все совершенные кубы от 1 до 1000000 и аппроксимировать кубический корень путем интерполяции между известными кубами. Значения кубов от 1 до 100 перечислены в таблице ниже.

    Учащимся рекомендуется тщательно запомнить эти значения кубов от 1 до 100 для более быстрого выполнения математических расчетов.

    В этом методе число умножается три раза (x × x × x), и полученное произведение дает нам куб этого числа. Например, куб числа 8 = 8 × 8 × 8 = 512. Здесь результирующее произведение «512» дает нам куб числа «8». Этот метод хорошо работает для небольших чисел.

    Сколько стоит куб от 1 до 100?

    Значение кубов от 1 до 100 – это список чисел, полученных путем трехкратного умножения целого числа (x 3 ). Это всегда будет положительное число для чисел от 1 до 100. 

    Какие существуют методы вычисления кубов от 1 до 100?

    Мы можем вычислить куб числа, используя повторяющееся умножение. Например, куб числа 3 можно вычислить, умножив 3 трижды (3 × 3 × 3).

    Если взять кубики от 1 до 100, сколько из них будет четных чисел?

    Четные числа от 1 до 100: 2, 4, 6, 8, 10, 12, 14, 16, 18, 20, 22, 24, 26, 28, 30, 32, 34, 36, 38, 40, 42, 44, 46, 48, 50, 52, 54, 56, 58, 60, 62, 64, 66, 68, 70, 72, 74, 76, 78, 80, 82, 84, 86, 88, 90, 92, 94, 96, 98, 100. Так как кубы четных чисел всегда четные. Следовательно, значение кубиков чисел 2, 4, 6, 8, 10, 12, 14, 16, 18, 20, 22, 24, 26, 28, 30, 32, 34, 36, 38, 40, 42, 44, 46, 48, 50, 52, 54, 56, 58, 60, 62, 64, 66, 68, 70, 72, 74, 76, 78, 80, 82, 84, 86, 88, 90, 92, 94, 96, 98 будут четными.

    Используя таблицу кубов от 1 до 100, найдите значение 10 плюс 10 куб плюс 20 куб.

    Значение 10 3 равно 1000, а 20 3 равно 8000. Таким образом, 10 + 10 3 + 20 3 = 10 + 1000 + 8000 = 10 + 1000 + 8000 = 9010 куб. плюс 20 кубов равно 9010.

    Какие значения кубов от 1 до 100 находятся в диапазоне от 1 до 1000 включительно?

    Значения кубов от 1 до 100 от 1 до 1000 равны 1 3 (1), 2 3 (8), 3 3 (27), 4 3 (64), 5 3 (125), 6 3 (216), 7 3 (3905), 8 3 (512), 9 3 (729) и 10 3 (1000).

    У 2х в квадрате 3: Mathway | Популярные задачи

    {2}+2 x-3}

    3-8 9 Оценить квадратный корень из 12 10 Оценить квадратный корень из 20 11 Оценить квадратный корень из 50 94 18 Оценить квадратный корень из 45 19 Оценить квадратный корень из 32 20 Оценить квадратный корень из 18 9{2}-4ac}}{2a}.

    Уравнения с корнями иррациональные: Иррациональные уравнения — что это, определение и ответ

    {2}\)

    Пример №1:

    Решим уравнение:

    \(\sqrt{3x} = 6\)

    1. Возведем обе части уравнения в квадрат, при условии, что они неотрицательные.

    \(\left\{ \begin{matrix} 3x \geq 0 \\ 6 \geq 0 \\ 3x = 36 \\ \end{matrix} \right.\ \)

    2. Определить знак числа справа можно сразу, 6 – положительное число, а значит больше нуля. В первом неравенстве выразим «х», получим:

    \(\left\{ \begin{matrix} x \geq 0 \\ 3x = 36 \\ \end{matrix} \right.\ \Longrightarrow \left\{ \begin{matrix} x \geq 0 \\ x = 12 \\ \end{matrix} \right.\ \)

    3. Система имеет решение при \(x = 12\). Запишем ответ.

    Ответ: 12.

    Если a < 0, то решений нет

    Например, решим уравнение:

    \(\sqrt{3x} = \ –6\)

    1. Составим систему:

    \(\left\{ \begin{matrix} 3x \geq 0 \\ –6 \geq 0 \\ 3x = 36 \\ \end{matrix} \right.\ \)

    2. Второе неравенство не имеет смысла, поэтому вся система не имеет решений.

    Ответ: \(\mathbf{\varnothing}\)

    То, что мы с вами сейчас сделали будет верно для любого корня четной степени. {n}\)

    Пример №2:

    Решим уравнение:

    \(\sqrt[3]{3x} = \ –6\)

    1. Видим корень нечетной степени – сразу возводим в эту степень обе части:

    \(\sqrt[3]{3x} = \ –6\)

    \(3x = \ –216\)

    \(x = \ –72\)

    2. Записываем ответ. Уравнение не имеет никаких ограничений.

    Ответ: –72.

    ВТОРОЙ ТИП ИРРАЦИОНАЛЬНЫХ УРАВНЕНИЙ «КОРЕНЬ=КОРЕНЬ»:

    \(\sqrt{f(x)} = \sqrt{g(x)}\)

    Решение:

    Если и слева и справа будет стоять корень алгоритм остается тот же: записываем ОДЗ и возводим обе части в квадрат.

    \(\sqrt{f(x)} = \sqrt{g(x)} \Leftrightarrow \left\{ \begin{matrix} f(x) \geq 0 \\ g(x) \geq 0 \\ f(x) = g(x) \\ \end{matrix} \right.\ \)

    Пример №3:

    Решим уравнение:

    \(\sqrt{–2x + 6} = \sqrt{15 + x}\)

    1. Составим систему:

    \(\left\{ \begin{matrix} –2x + 6 \geq 0 \\ 15 + x \geq 0 \\ –2x + 6 = 15 + x \\ \end{matrix} \right.\ \)

    2. {2}\ –\ 20x + 36 = 0 \\ \end{matrix} \right.\ \)

    5. Решим квадратное уравнение через теорему Виета:

    \(\left\lbrack \frac{x_{1} = 18}{x_{2} = 2} \right.\ \)

    Только \(x = 2\) является уравнением системы. Это значение переменной и запишем в ответ.

    Ответ: 2.

    ПЯТЫЙ ВИД ИРРАЦИОНАЛЬНОГО УРАВНЕНИЯ «КОРЕНЬ – КОРЕНЬ = ЧИСЛО»:

    \(\sqrt{f(x)}\ –\ \sqrt{g(x)} = a\)

    Решение:

    \(\sqrt{f(x)} = a + \sqrt{g(x)}\)

    И решаем такое уравнение как четвертый вид «корень + корень = число».

    Пример решения иррационального уравнения с двумя корнями

    Нам нужно решить иррациональное уравнение (см. что такое иррациональное уравнение). В его записи присутствуют два корня и еще одно слагаемое помимо них. Такие иррациональные уравнения очень характерны, для их решения обычно используется метод возведения обеих частей уравнения в одну и ту же степень. Причем, для избавления от обоих радикалов к возведению обеих частей уравнения в степень придется прибегать два раза.

    Напомним последовательность действий для решения иррациональных уравнений по методу возведения обеих частей в одну и ту же степень:

    • Во-первых, переходим к более простому уравнению, для чего циклически выполняем следующие три действия:
      • Уединяем радикал.
      • Возводим обе части полученного уравнения в одну и ту же натуральную степень.
      • Упрощаем вид уравнения, полученного после возведения в степень.
    • Во-вторых, решаем полученное уравнение.
    • В-третьих, отсеиваем посторонние корни, если выше проводилось возведение в четную степень.

    Начнем. Выполним тройку действий – уединение радикала, возведение в степень, упрощение вида – в первый раз.

    Уединение радикала приводит нас к уравнению .

    Так как степень уединенного корня равна двум, то возведем обе части уравнения во вторую степень: , что дальше позволит избавиться от уединенного радикала.

    Теперь упростим вид полученного уравнения при помощи преобразования уравнений. В первую очередь, базируясь на определении корня, заменим выражение в левой части тождественно равным выражением x−6, и, учитывая формулу сокращенного умножения «квадрат разности», заменим выражение в правой части тождественно равными ему выражением . Имеем . Продолжим упрощать вид уравнения. Вновь оттолкнемся от определения корня для замены выражения тождественно равным ему выражением x+2, а числовое выражение 22 заменим его значением четыре: . Дальнейшие преобразования не нуждаются в комментариях:

    Очевидно, после первого прохода цикла мы освободились от одного корня, но остался еще один корень. Поэтому второй раз выполним указанную тройку действий – уединение радикала, возведение обеих частей уравнения в степень, упрощение выражения.

    В уравнении уединять радикал не нужно, так как это уже сделано.

    Для избавления от квадратного корня выполним возведение обеих частей уравнения в квадрат: .

    Упрощаем вид полученного уравнения:
    x+2=9,
    x=7.

    Так мы получили тривиальное уравнение. На этом первый этап решения по методу возведения обеих частей уравнения в одну и ту же степень завершен. Переходим ко второму этапу.

    Второй этап – решение полученного уравнения – также можно считать завершенным, так как корень уравнения x=7 очевиден. Это число 7.

    Остается третий этап решения – отсеивание посторонних корней. В нашем случае отсеивание обязательно, так как некоторые из проводимых выше преобразований могли привести к появлению посторонних корней. Действительно, мы дважды прибегали к возведению обеих частей уравнения в одну и ту же четную степень, а, как известно, такое преобразование может привести к появлению посторонних корней. Также в цепочке преобразований был переход от уравнения к уравнению x+2=9, при котором расширилась ОДЗ, что тоже могло привести к появлению посторонних корней. Так что проведем отсеивание посторонних корней. Сделаем это через проверку подстановкой. Подставим найденный корень в иррациональное уравнение , имеем

    Подстановка дала верное числовое равенство, значит, x=7 является искомым корнем.

    На этом решение иррационального уравнения методом возведения обеих частей уравнения в одну и ту же степень завершено, оно потребовало двух возведений в степень для избавления от двух корней.

    Приведем краткий вариант решения:

    Алгебра — уравнения с радикалами

    Онлайн-заметки Пола
    Главная / Алгебра / Решение уравнений и неравенств / Уравнения с радикалами

    Показать мобильное уведомление Показать все примечания Скрыть все примечания

    Мобильное уведомление

    Похоже, вы используете устройство с «узкой» шириной экрана ( т. е. вы, вероятно, используете мобильный телефон). Из-за характера математики на этом сайте лучше всего просматривать в ландшафтном режиме. Если ваше устройство не находится в ландшафтном режиме, многие уравнения будут отображаться сбоку вашего устройства (должна быть возможность прокрутки, чтобы увидеть их), а некоторые пункты меню будут обрезаны из-за узкой ширины экрана.

    Раздел 2.10: Уравнения с радикалами

    Название этого раздела может немного ввести в заблуждение. Название, кажется, подразумевает, что мы будем рассматривать уравнения, в которых есть радикалы. Однако мы собираемся ограничиться уравнениями с квадратными корнями. Методы, которые мы собираемся применить здесь, можно использовать для решения уравнений с другими радикалами, однако работа обычно значительно сложнее, чем при работе с квадратными корнями. Поэтому в этом разделе мы будем работать только с квадратными корнями.

    Прежде чем продолжить, следует также упомянуть, что в некоторых учебниках по алгебре вы найдете этот раздел с уравнениями, приводимыми к квадратичной форме. Причина в том, что в большинстве случаев мы на самом деле приходим к решению квадратного уравнения. Однако подход существенно отличается, поэтому в этом курсе мы разделим две темы на разные разделы.

    Обычно лучше всего посмотреть, как они работают, на примере.

    Пример 1 Решить \(x = \sqrt {x + 6} \).

    Показать решение

    В этом уравнении основной проблемой является квадратный корень. Если бы этого не было, мы могли бы решить проблему. Весь процесс, который мы собираемся пройти здесь, настроен на устранение квадратного корня. Однако, как мы увидим, шаги, которые мы собираемся предпринять, на самом деле могут вызвать у нас проблемы. Итак, давайте посмотрим, как это все работает. 92} — x — 6 & = 0\\ \left( {x — 3} \right)\left( {x + 2} \right) & = 0\hspace{0.25in}\hspace{0.25in} \Rightarrow \hspace{0,25 дюйма}\hspace{0,25 дюйма}x = 3,\,\,\,x = — 2\end{align*}\]

    После возведения в квадрат обеих частей мы видим, что мы получаем факторизуемое квадратное уравнение, которое дает нам два решения \(x = 3\) и \(x = — 2\).

    Теперь, без всякой видимой причины, давайте сделаем то, чего мы фактически не делали со времен раздела о решении линейных уравнений. Давайте проверим наши ответы. Помните также, что нам нужно проверить ответы в исходном уравнении! Это очень важно. 9? \sqrt { — 2 + 6} \\ — 2 & \ne \sqrt 4 = 2\hspace{0,25 дюйма}\hspace{0,25 дюйма}{\mbox{НЕ ОК}}\end{align*}\]

    У нас проблема. Напомним, что квадратные корни ВСЕГДА положительны, поэтому \(x = — 2\) не работает в исходном уравнении. Одна из возможностей здесь заключается в том, что мы где-то допустили ошибку. Однако мы можем вернуться и посмотреть, и мы быстро увидим, что не ошиблись.

    Итак, в чем дело? Помните, что нашим первым шагом в процессе решения было возведение обеих сторон в квадрат. Заметьте, что если мы подставим \(x = — 2\) в квадратное уравнение, которое мы решили, оно на самом деле будет его решением. Когда мы возвели в квадрат обе части уравнения, мы фактически изменили уравнение и в процессе ввели решение, которое не является решением исходного уравнения.

    При таких проблемах жизненно важно проверить свои решения, так как это часто случается. В этом случае мы берем только те значения, которые являются фактическими решениями исходного уравнения.

    Итак, исходное уравнение имело единственное решение \(x = 3\).

    Теперь, как показал этот пример, мы должны быть очень осторожны при решении этих уравнений. Когда мы решим квадратное уравнение, мы получим два решения, и возможно, что оба из них, одно из этих или ни одно из этих значений не являются решениями исходного уравнения. Единственный способ узнать это проверить свои решения!

    Давайте поработаем еще с парой примеров, которые немного сложнее.

    Пример 2. Решите каждое из следующих уравнений.

    1. \(у + \sqrt {у — 4} = 4\)
    2. \(1 = t + \sqrt {2t — 3} \)
    3. \(\sqrt {5z + 6} — 2 = z\)

    Показать все решения Скрыть все решения

    a \(y + \sqrt {y — 4} = 4\) Показать решение

    В этом случае давайте заметим, что если мы просто возведем в квадрат обе стороны, у нас будут проблемы. 92}\]

    с \(a = y\) и \(b = \sqrt {y — 4} \). Вы должны быть в состоянии сделать это, потому что, хотя это, возможно, не сработало здесь, нам понадобится такая работа в следующем наборе задач.

    В чем проблема? Хорошо помните, что смысл возведения в квадрат обеих сторон в первой задаче заключался в том, чтобы исключить квадратный корень. Мы этого не сделали. В задаче по-прежнему есть квадратный корень, и мы сделали оставшуюся часть задачи еще более беспорядочной. 92} — 9y + 20\\ 0 & = \left( {y — 5} \right)\left( {y — 4} \right)\hspace{0.25in} \Rightarrow \hspace{0.25in}y = 4 ,\,\,\,y = 5\end{align*}\]

    Как и в первом примере, нам нужно убедиться и проверить оба этих решения. Опять же, убедитесь, что вы проверили исходное уравнение. Как только мы возведем в квадрат обе стороны, мы изменим задачу, и поэтому проверка здесь не принесет нам никакой пользы. На самом деле проверка там вполне может привести нас к неприятностям.

    9? 4\\ 6 & \ne 4\hspace{0,25 дюйма}{\mbox{НЕ}}{\mbox{OK}}\end{align*}\]

    Итак, как и в первом примере, который мы рассмотрели, на самом деле существует единственное решение исходного уравнения \(y = 4\). 2} — z — 2\\ & 0 = \left( {z — 2 } \right)\left( {z + 1} \right)\hspace{0.25in} \Rightarrow \hspace{0.25in}\,\,\,\,\,z = — 1,\,\,\, \,z = 2\конец{выравнивание*}\] 9? 2\\ 4 — 2 & = 2\hspace{0,25 дюйма}{\mbox{OK}}\end{align*}\]

    Это тоже было решением.

    Итак, в данном случае мы увидели пример, в котором оба возможных решения фактически также являются решениями исходного уравнения.

    Итак, как мы видели в предыдущем наборе примеров, как только мы получим список возможных решений, от ни одного до всех, они могут быть решениями исходного уравнения. Не забывайте проверять свои ответы!

    Хорошо, давайте поработаем еще с одним набором примеров, которые имеют дополнительную сложность. До сих пор все уравнения, которые мы рассматривали, содержали один квадратный корень. Однако в этих уравнениях может быть более одного квадратного корня. Следующий набор примеров предназначен для того, чтобы показать нам, как справляться с такого рода проблемами.

    Пример 3. Решите каждое из следующих уравнений.

    1. \(\sqrt {2x — 1} — \sqrt {x — 4} = 2\)
    2. \(\sqrt {t + 7} + 2 = \sqrt {3 — t} \)

    Показать все решения Скрыть все решения

    Показать обсуждение

    В обоих из них есть два квадратных корня в задаче. Однако мы будем работать с ними в основном таким же образом. Первый шаг состоит в том, чтобы получить один из квадратных корней в одной части уравнения, а затем возвести в квадрат обе части. На этом этапе процесс отличается, поэтому мы увидим, как двигаться дальше, как только мы достигнем его в первом примере. 92}\\ 2x — 1 & = 4 + 4\sqrt {x — 4} + x — 4\\ 2x — 1 & = 4\sqrt {x — 4} + x\end{align*}\]

    Итак, у нас все еще есть квадратный корень в задаче, но нам удалось исключить один из них. Не только это, но и то, что у нас осталось, идентично примерам, с которыми мы работали в первой части этого раздела. 2} — 18x + 65 & = 0 \\ \left( {x — 13} \right)\left( {x — 5} \right) & = 0\hspace{0.25in} \Rightarrow \hspace{0.25in}x = 13,\,\,\ ,х = 5\конец{выравнивание*}\] 9? \sqrt 9 \\ 1 + 2 & = 3\hspace{0,25 дюйма}{\mbox{OK}}\end{align*}\]

    Похоже, в этом случае у нас есть единственное решение, \(t = — 6\).

    Итак, когда в задаче больше одного квадратного корня, мы снова сталкиваемся с задачей проверки наших возможных решений. Вполне возможно, что все возможные решения, от нуля до всех, на самом деле будут решениями, и единственный способ узнать наверняка — проверить их в исходном уравнении.

    10.7: Решение радикальных уравнений — Математика LibreTexts

    1. Последнее обновление
    2. Сохранить как PDF
  • Идентификатор страницы
    49983
  • Цели обучения

    К концу этого раздела вы сможете: 9{2}−6n+8=0\).
    Если вы пропустили эту проблему, просмотрите пример 6.45.

    Решение подкоренных уравнений

    В этом разделе мы будем решать уравнения, в которых подкоренное выражение содержит переменную. Уравнение этого типа называется радикальным уравнением .

    Определение: подкоренное уравнение

    Уравнение, в котором переменная стоит под корнем подкоренного выражения, называется подкоренным уравнением .

    Как обычно, при решении этих уравнений то, что мы делаем с одной частью уравнения, мы должны делать и с другой его частью. Как только мы изолируем радикал, наша стратегия будет состоять в том, чтобы возвести обе части уравнения в степень индекса. Это устранит радикал. 9{п}=а\).

    Пример \(\PageIndex{1}\) решения радикального уравнения

    Решите: \(\sqrt{5 n-4}-9=0\).

    Решение :

    Шаг 1 : Выделите радикал на одной стороне уравнения.

    Чтобы изолировать радикал, добавьте \(9\) к обеим сторонам.

    Упростить.

    \(\ begin{array}{c}{\sqrt{5 n-4}-9=0} \\ {\sqrt{5 n-4}-9\color{red}{+9{2}=а\). \(\begin{выровнено} 5 n-4 &=81 \\ 5 n &=85 \\ n &=17 \end{выровнено}\)
    Шаг 4 : Проверьте ответ в исходном уравнении.  

    Проверьте ответ.

    \(\ begin{array}{r}{\sqrt{5 n-4}-9=0} \\ {\sqrt{5(\color{red}{17}\color{black}{)}- 4}-9 \stackrel{?}{=} 0} \\ {\sqrt{85-4}-9 \stackrel{?}{=} 0} \\ {\sqrt{81}-9 \stackrel{? }{=} 0} \\ {9-9=0} \\ {0=0}\конец{массив}\)

    Решение: \(n=17\).

    Таблица 8.6.1
    Упражнение \(\PageIndex{1}\)

    Решите: \(\sqrt{3 m+2}-5=0\).

    Ответить

    \(м=\фракция{23}{3}\)

    Упражнение \(\PageIndex{2}\)

    Решите: \(\sqrt{10 z+1}-2=0\).

    Ответить

    \(z=\frac{3}{10}\)

    Решите радикальное уравнение с одним радикалом

    1. Изолируйте радикал с одной стороны уравнения.
    2. Возведите обе части уравнения в степень индекса.
    3. Решите новое уравнение.
    4. Проверьте ответ в исходном уравнении.

    Когда мы используем радикальный знак, он указывает на главный или положительный корень. Если уравнение имеет радикал с четным индексом, равным отрицательному числу, то это уравнение не будет иметь решения.

    Пример \(\PageIndex{2}\)

    Решите: \(\sqrt{9 k-2}+1=0\).

    Решение :

     
    Чтобы изолировать радикал, вычтите \(1\) с обеих сторон.
    Упрощение.
    Таблица 8. 6.2

    Поскольку квадратный корень равен отрицательному числу, уравнение не имеет решения.

    Упражнение \(\PageIndex{3}\)

    Решите: \(\sqrt{2 r-3}+5=0\).

    Ответить

    нет решения

    Упражнение \(\PageIndex{4}\)

    Решите: \(\sqrt{7 s-3}+2=0\).

    Ответить

    нет решения

    Если одна часть уравнения с квадратным корнем является двучленом, мы используем образец произведения биномиальных квадратов, когда возводим его в квадрат. 9{2}}\end{array}\)

    Не забудьте средний термин!

    Пример \(\PageIndex{3}\)

    Решить: \(\sqrt{p-1}+1=p\).

    Решение :

     
    Чтобы изолировать радикал, вычтите \(1\) с обеих сторон.
    Упрощение.
    Возведите в квадрат обе части уравнения.
    Упростите, используя образец произведения биномиальных квадратов справа. Затем решите новое уравнение.
    Это квадратное уравнение, поэтому с одной стороны получаем ноль.
    Фактор правой стороны.
    Использовать свойство нулевого продукта.
    Решите каждое уравнение.
    Проверьте ответы.  
     
    Таблица 8.6.3

    Решения: \(p=1, p=2\).

    Упражнение \(\PageIndex{5}\)

    Решите: \(\sqrt{x-2}+2=x\).

    Ответить

    \(х=2, х=3\)

    9{3}\) Упрощение. \(5 х+1=-64\) Решите уравнение. \(5 х=-65\)   \(х=-13\) Проверьте ответ.       Решение: \(x=-13\). Таблица 8.6.4
    Упражнение \(\PageIndex{7}\)

    Решить: \( \sqrt[3]{4 x-3}+8=5\)

    Ответ

    \(х=-6\)

    Упражнение \(\PageIndex{8}\)

    Решить: \(\sqrt[3]{6 x-10}+1=-3\)

    Ответ

    \(х=-9\)

    Иногда уравнение может содержать рациональные показатели вместо радикала. Мы используем те же методы для решения уравнения, что и в случае, когда у нас есть радикал. Возведем каждую часть уравнения в степень знаменателя рационального показателя. {m \cdot n}\), мы имеем, например, 9{4}\) Упрощение. \(3 х-2=16\) Решите уравнение. \(3x=18\)   \(х=6\) Проверьте ответ.       Решение: \(x=6\). 9{\frac{1}{4}}+5=7\)

    Ответ

    \(х=6\)

    Иногда решение радикального уравнения приводит к двум алгебраическим решениям, но одно из них может быть посторонним решением !

    Пример \(\PageIndex{6}\)

    Решите: \(\sqrt{r+4}-r+2=0\).

    Решение :

    {2}-5 р\)
      \(\sqrt{r+4}-r+2=0\)
    Фактор правой стороны. \(0=r(r-5)\)
    Использовать свойство нулевого продукта. \(0=r \quad 0=r-5\)
    Решите уравнение. \(r=0 \квадратный r=5\)
    Проверьте свой ответ.  
    Решение: \(r=5\).
      \(r=0\) — экстравагантное решение.
    Таблица 8.6.6
    Упражнение \(\PageIndex{11}\)

    Решить: \(\sqrt{m+9}-m+3=0\)

    Ответ

    \(м=7\)

    Упражнение \(\PageIndex{12}\)

    Решите: \(\sqrt{n+1}-n+1=0\).

    Ответить

    \(n=3\)

    Когда перед корнем стоит коэффициент, мы должны возвести и его в степень индекса.

    Пример \(\PageIndex{7}\)

    Решите: \(3 \sqrt{3 x-5}-8=4\).

    Решение :

      \(3 \sqrt{3 x-5}-8=4\)
    Изолируйте радикальный термин. \(3 \sqrt{3 x-5}=12\)
    Изолируйте радикал, разделив обе части на \(3\). 9{2}\)
    Упростите, затем решите новое уравнение. \(3 х-5=16\)
      \(3x=21\)
    Решите уравнение. \(х=7\)
    Проверьте ответ.  
     
      Решение: \(x=7\).
    Таблица 8.6.7
    Упражнение \(\PageIndex{13}\)

    Решите: \(2 \sqrt{4 a+4}-16=16\).

    Ответить

    \(а=63\)

    Упражнение \(\PageIndex{14}\)

    Решить: \(3 \sqrt{2 b+3}-25=50\)

    Ответ

    \(б=311\)

    Решение радикального уравнения с двумя радикалами

    Если радикальное уравнение имеет два радикала, мы начинаем с выделения одного из них. Часто проще всего сначала выделить более сложный радикал. 9{3}\)

    Упростите, затем решите новое уравнение.

    \(\begin{align} 4 x-3 &=3 x+2 \\ x-3 &=2 \\ x &=5 \end{align}\)

    Решение \(x=5 \).

    Проверьте ответ.

    Мы предоставляем вам показать, что \(5\) проверяет!

    Упражнение \(\PageIndex{15}\)

    Решите: \(\sqrt[3]{5 x-4}=\sqrt[3]{2 x+5}\).

    Ответить

    \(х=3\)

    Упражнение \(\PageIndex{16}\)

    Решите: \(\sqrt[3]{7 x+1}=\sqrt[3]{2 x-5}\).

    Ответить

    \(х=-\фракция{6}{5}\)

    Иногда после возведения обеих частей уравнения в степень у нас все еще есть переменная внутри корня. Когда это происходит, мы повторяем Шаг 1 и Шаг 2 нашей процедуры. Мы изолируем радикал и снова возводим обе части уравнения в степень индекса.

    Пример \(\PageIndex{9}\) как решить радикальное уравнение

    Решите: \(\sqrt{m}+1=\sqrt{m+9}\).

    Решение :

    Шаг 1 : Выделите один из радикальных членов на одной стороне уравнения. Радикал справа изолирован. \(\sqrt{м}+1=\sqrt{м+9}\)
    Шаг 2 : Возведите обе части уравнения в степень индекса.

    Подравниваем обе стороны. 9{2}\)

    Шаг 3 : Есть еще радикалы? Если да, повторите Шаг 1 и Шаг 2 еще раз.

    Если нет, решите новое уравнение.

    В уравнении все еще есть радикал.

    Итак, мы должны повторить предыдущие шаги. Выделите корневой термин.

    Здесь мы можем легко выделить радикал, разделив обе части на \(2\). 9{2} \\ m &=16 \end{выровнено}\)

    Шаг 4 : Проверьте ответ в исходном уравнении.  

    \(\begin{align}\sqrt{m}+1&=\sqrt{m+9} \\ \sqrt{\color{red}{16}}\color{black}{+}1& \stackrel{? }{=} \sqrt{\color{red}{16}\color{black}{+}9} \\ 4+1& \stackrel{?}{=} 5 \\ 5&=5\end{выровнено}\ )

    Решение: \(m=16\).

    Таблица 8.6.8
    Упражнение \(\PageIndex{17}\)

    Решите: \(3-\sqrt{x}=\sqrt{x-3}\).

    Ответить

    \(х=4\)

    Упражнение \(\PageIndex{18}\)

    Решите: \(\sqrt{x}+2=\sqrt{x+16}\).

    Ответить

    \(х=9\)

    Здесь мы суммируем шаги. Мы скорректировали наши предыдущие шаги, чтобы включить в уравнение более одного радикала. Теперь эта процедура будет работать для любых радикальных уравнений. 9{2}\).

    Пример \(\PageIndex{10}\)

    Решите: \(\sqrt{q-2}+3=\sqrt{4 q+1}\).

    Решение :

     
    Радикал справа изолирован. Подровняйте обе стороны.
    Упрощение.
    В уравнении все еще есть радикал, поэтому мы должны повторить предыдущие шаги. Изолировать радикал.
    Квадрат с обеих сторон. Разделение обеих частей на \(6\) не помогло бы. Не забудьте возвести в квадрат как \(6\), так и \(\sqrt{q-2}\).
    Упростите, затем решите новое уравнение.
    Распределить.
    Это квадратное уравнение, поэтому с одной стороны получаем ноль.
    Фактор правой стороны.
    Использовать свойство нулевого продукта.
    Чеки оставляются вам. Решения: \(q=6\) и \(q=2\).
    Таблица 8.6.9
    Упражнение \(\PageIndex{19}\)

    Решить: \(\sqrt{x-1}+2=\sqrt{2 x+6}\)

    Ответ

    \(х=5\)

    Упражнение \(\PageIndex{20}\)

    Решить: \(\sqrt{x}+2=\sqrt{3 x+4}\)

    Ответ

    \(х=0 х=4\)

    Использование радикалов в приложениях

    По мере прохождения курсов в колледже вы столкнетесь с формулами, содержащими радикалы, во многих дисциплинах. Мы немного изменим нашу стратегию решения задач для приложений геометрии, чтобы получить план решения приложений с формулами из любой дисциплины.

    Используйте стратегию решения проблем для приложений с формулами

    1. Прочтите задачу и убедитесь, что все слова и идеи понятны. При необходимости нарисуйте рисунок и подпишите его с помощью данной информации.
    2. Определите , что мы ищем.
    3. Назовите то, что мы ищем, выбрав переменную для ее представления.
    4. Переведите в уравнение, написав соответствующую формулу или модель для данной ситуации. Замените предоставленную информацию.
    5. Решите уравнение , используя хорошие методы алгебры.
    6. Проверьте ответ в задаче и убедитесь, что он имеет смысл.
    7. Ответьте на вопрос полным предложением.

    Одно из применений радикалов связано с влиянием гравитации на падающие предметы. Формула позволяет определить, через какое время упавший предмет ударится о землю.

    Определение: Падающие предметы

    На Земле, если объект падает с высоты \(h\) футов, время в секундах, необходимое для достижения земли, определяется по формуле

    \(t=\frac{\sqrt{ h}}{4}\)

    Например, если объект падает с высоты \(64\) футов, мы можем найти время, необходимое для достижения земли, подставив \(h=64\) в формула.

     
     
    Извлеките квадратный корень из \(64\).
    Упростите дробь.
    Таблица 8.6.10

    Предмету, упавшему с высоты \(64\) футов, потребуется \(2\) секунды, чтобы достичь земли.

    Пример \(\PageIndex{11}\)

    Марисса уронила солнцезащитные очки с моста \(400\) футов над рекой. Используйте формулу \(t=\frac{\sqrt{h}}{4}\), чтобы найти, сколько секунд потребовалось солнцезащитным очкам, чтобы достичь реки.

    Решение :

    Шаг 1 : Прочтите задачу.  
    Шаг 2 : Определите что мы ищем. Время, за которое солнечные очки достигают реки.
    Шаг 3 : Назовите то, что мы ищем. Пусть (t=\) время.
    Шаг 4 : Переведите в уравнение, написав соответствующую формулу. Замените предоставленную информацию.
    Шаг 5 : Решите уравнение .
     
    Шаг 6 : Проверьте ответ в задаче и убедитесь, что он имеет смысл.
    \(5\) секунд кажется разумным отрезком времени? Да.
    Шаг 7 : Ответьте на уравнение. Солнцезащитные очки доберутся до реки через \(5\) секунд.
    Таблица 8. 6.11
    Упражнение \(\PageIndex{21}\)

    Вертолет сбросил спасательный пакет с высоты \(1296\) футов. Используйте формулу \(t=\frac{\sqrt{h}}{4}\), чтобы найти, сколько секунд потребовалось пакету, чтобы достичь земли.

    Ответить

    \(9\) секунд

    Упражнение \(\PageIndex{22}\)

    Мойщик окон уронил швабру с платформы \(196\) футов над тротуаром. Используйте формулу \(t=\frac{\sqrt{h}}{4}\), чтобы найти, сколько секунд потребовалось, чтобы швабра достигла тротуара.

    Ответить

    \(3,5\) секунд

    Полицейские, расследующие автомобильные аварии, измеряют длину следов заноса на тротуаре. Затем они используют квадратные корни, чтобы определить скорость , в милях в час, автомобиль ехал до того, как затормозил.

    Определение: следы заноса и скорость автомобиля

    Если длина следов заноса составляет \(d\) футов, то скорость \(s\) автомобиля до включения тормозов можно найти по формуле используя формулу

    \(s=\sqrt{24 d}\)

    Пример \(\PageIndex{12}\)

    После автомобильной аварии следы заноса одного автомобиля измерялись \(190\) футов. Используйте формулу \(s=\sqrt{24d}\), чтобы найти скорость автомобиля до включения тормозов. Округлите ответ до десятых.

    Решение :

    Шаг 1 : Прочтите задачу.  
    Шаг 2 : Определите , что мы ищем. Скорость автомобиля.
    Шаг 3 : Имя то, что мы ищем. Пусть \(s=\) скорость.
    Шаг 4 : Переведите в уравнение, написав соответствующую формулу. Замените предоставленную информацию.
    Шаг 5 : Решите уравнение .
     
    Округлить до \(1\) десятичного знака.
     
      Скорость автомобиля до торможения составляла \(67,5\) миль в час.
    Таблица 8.6.12
    Упражнение \(\PageIndex{23}\)

    Исследователь ДТП измерил следы заноса автомобиля. Длина следов заноса составляла \(76\) футов. Используйте формулу \(s=\sqrt{24d}\), чтобы найти скорость автомобиля до включения тормозов. Округлите ответ до десятых.

    Ответить

    \(42,7\) миль/ч

    Упражнение \(\PageIndex{24}\)

    Следы заноса автомобиля, попавшего в аварию, были \(122\) футов в длину. Используйте формулу \(s=\sqrt{24d}\), чтобы найти скорость транспортного средства до включения тормозов. Округлите ответ до десятых.

    Ответить

    \(54,1\) миль/ч

    Получите доступ к этим онлайн-ресурсам для получения дополнительных инструкций и практики решения радикальных уравнений. 9{2}}\конец{массив}\)

  • Решение радикального уравнения
    1. Изолируйте один из радикальных членов на одной стороне уравнения.
    2. Возведите обе части уравнения в степень индекса.
    3. Есть еще радикалы?
      Если да, повторите Шаг 1 и Шаг 2 еще раз.
      Если нет, решите новое уравнение.
    4. Проверьте ответ в исходном уравнении.
  • Стратегия решения проблем для приложений с формулами
    1. Прочитайте задачу и убедитесь, что все слова и идеи понятны. При необходимости нарисуйте рисунок и подпишите его с помощью данной информации.
    2. Определите, что мы ищем.
    3. Назовите то, что мы ищем, выбрав переменную для ее представления.
    4. Переведите в уравнение, написав соответствующую формулу или модель для данной ситуации. Замените предоставленную информацию.
    5. Решите уравнение, используя хорошие методы алгебры.
    6. Проверьте ответ в задаче и убедитесь, что он имеет смысл.
    7. Ответьте на вопрос полным предложением.
  • Падающие предметы
    • На Земле, если объект падает с высоты \(h\) футов, время в секундах, необходимое для достижения земли, определяется по формуле \(t=\frac{\sqrt{h} {4}\).
  • Следы заноса и скорость автомобиля
    • Если длина следов заноса составляет \(d\) футов, то скорость \(s\) автомобиля до включения тормозов можно найти по формуле \(s=\sqrt{24d} \).
  • Глоссарий

    радикальное уравнение
    Уравнение, в котором переменная стоит под корнем подкоренного выражения, называется подкоренным уравнением.

    10.7: Solve Radical Equations распространяется по незаявленной лицензии и был создан, изменен и/или курирован LibreTexts.

    Как делать разложение на множители: Разложение на множители — урок. Алгебра, 7 класс.

    § Способ группировки. Разложение многочлена на множители.

    Что такое многочлен. Степень многочлена Стандартный вид многочлена. Приведение подобных Сложение и вычитание многочленов Умножение многочлена на одночлен Умножение многочлена на многочлен Деление многочлена на одночлен Вынесение общего множителя за скобки Способ группировки

    Кроме вынесения общего множителя за скобки существует еще один способ разложения многочлена на множители — способ группировки.

    Этот способ разложения на множители считается более сложным, поэтому перед его изучением, убедитесь, что вы уверенно выносите общий множитель за скобки.

    Запомните!

    Чтобы разложить многочлен на множители способом группировки, необходимо сделать следующее.

    1. Подчеркнуть повторяющиеся буквы и записать друг за другом одночлены с одинаковыми буквенными множителями.
    2. Вынести общий множитель за скобки у каждой группы одночленов.
    3. Вынести полученный общий многочлен за скобки.

    Рассмотрим пример разложения многочлена на множители способом группировки.

    1. Подчеркнем повторяющиеся буквенные множители в одночленах.
    2. У нас получится две группы одночленов с повторяющимися буквенными множителями.
    3. Вынесем общий множитель за скобки у каждой группы одночленов.
    4. Проверим, верно ли мы вынесли общий множитель за скобки. Для этого раскроем скобки обратно. Мы получили исходный многочлен, значит, мы правильно вынесли общий множитель за скобки.
    5. Теперь в полученном результате вынесем общий многочлен «(a + b)» за скобки.

    Группировать одночлены можно по-разному. При правильной группировке должен появиться общий многочлен.

    Рассмотрим пример. Требуется разложить многочлен на множители, используя способ группировки.

    Первый способ

    48xz2 + 32xy2 − 15z2 − 10y2 =

    Обратим внимание, что в двух одночленах повторяется «y2» и «z2». Подчеркнем повторяющиеся одночлены и запишем их друг за другом. Затем вынесем общий множитель у каждой группы одночленов.

    48xz2 + 32xy2 − 15z2 − 10y2 = 48xz2 − 15z2 + 32xy2 − 10y2 = 3z2(16x − 5) + 2y2(16x − 5) =
    = (16x − 5)(3z2 + 2y2)

    Второй способ

    Запишем пример еще раз. Теперь обратим внимание, что в первых двух одночленах повторяется «x». Подчеркнем повторяющиеся одночлены. Вынесем общий множитель у каждой группы одночленов.

    48xz2 + 32xy2 − 15z2 − 10y2 = 16x(3z2 + 2y2) − 5(3z2 + 2y2) = (3z2 + 2y2)(16x − 5)

    В итоге получился такой же ответ, как и при первом способе.

    Рассмотрим еще один пример разложения многочлена способом группировки.

    • 4q(p − 1) + p − 1 = 4q(p − 1) + (p − 1) = 4q(p − 1) + 1 · (p − 1) = (p − 1)(4q + 1)
      В этом примере следует отметить, что для вынесения общего многочлена мы добавили умножение на 1 к многочлену (p − 1), что не изменяет результат умножения.
      Это помогает понять, что останется во второй скобке после вынесения общего многочлена.

    Смена знаков в скобках

    Важно!

    Иногда для вынесения общего многочлена требуется сменить все знаки одночленов в скобках на противоположные.

    Для этого за скобки выносится знак «−», а в скобках у всех одночленов меняются знаки на противоположные.

    2ab2 − 3x + 1 = −(−2ab2 + 3x − 1)

    Рассмотрим пример способа группировки, где для вынесения общего многочлена, нам потрубуется выполнить смену знаков в скобках.

    • 2m(m − n) + n − m = −2m( −m + n) + (n − m) = −2m(n − m) + 1 · (n − m) =
      = (n − m)(−2m + 1)

    Что такое многочлен. Степень многочлена Стандартный вид многочлена. Приведение подобных Сложение и вычитание многочленов Умножение многочлена на одночлен Умножение многочлена на многочлен Деление многочлена на одночлен Вынесение общего множителя за скобки Способ группировки


    Ваши комментарии

    Важно!

    Чтобы оставить комментарий, вам нужно войти на наш сайт при помощи «ВКонтакте».

    Оставить комментарий:

    Отправить

    Разложение многочлена на множители: примеры, правило

    Для того, чтобы разложить на множители, необходимо упрощать выражения. Это необходимо для того, чтобы можно было в дальнейшем сократить. Разложение многочлена имеет смысл тогда, когда его степень не ниже второй. Многочлен с первой степенью называют линейным.

    Статья раскроет все понятия разложения, теоретические основы и способы разложений многочлена на множители.

    Теория

    Теорема 1

    Когда любой многочлен со степенью n, имеющие вид Pnx=anxn+an-1xn-1+…+a1x+a0 , представляют в виде произведения с постоянным множителем со старшей степенью an и n линейных множителей (x-xi) , i=1, 2, …, n, тогда Pn(x)=an(x-xn)(x-xn-1)·…·(x-x1) , где xi , i=1, 2, …, n – это и есть корни многочлена.

    Теорема предназначена для корней комплексного типа xi ,i=1, 2, …, n и для комплексных коэффициентов ak ,k=0, 1, 2, …, n. Это и есть основа любого разложения.

    Когда коэффициенты вида ak, k=0, 1, 2, …, n являются действительными числами, тогда комплексные корни, которые будут встречаться сопряженными парами. Например, корни x1  и x2 , относящиеся к многочлену вида Pnx=anxn+an-1xn-1+…+a1x+a0  считаются комплексно сопряженным, тогда другие корни являются действительными, отсюда получаем, что многочлен примет вид Pn(x)=an(x-xn)(x-xn-1)·. ..·(x-x3)x2+px+q , где x2+px+q=(x-x1)(x-x2).

    Замечание

    Корни многочлена могут повторяться. Рассмотрим доказательство теоремы алгебры, следствия из теоремы Безу.

    Основная теорема алгебры

    Теорема 2

    Любой многочлен со степенью n имеет как минимум один корень.

    Теорема Безу

    После того, как произвели деление многочлена вида Pnx=anxn+an-1xn-1+…+a1x+a0  на (x-s), тогда получаем остаток, который равен многочлену в точке s, тогда получим

    Pnx=anxn+an-1xn-1+…+a1x+a0=(x-s)·Qn-1(x)+Pn(s) , где Qn-1(x)  является многочленом со степенью n-1.

    Следствие из теоремы Безу

    Когда корень многочлена Pn(x) считается s, тогда Pnx=anxn+an-1xn-1+…+a1x+a0=(x-s)·Qn-1(x) . Данное следствие является достаточным при употреблении для описания решения.

    Разложение на множители квадратного трехчлена

    Квадратный трехчлен вида ax2+bx+c  можно разложить на линейные множители. тогда получим, что ax2+bx+c=a(x-x1)(x-x2), где x1 и x2  — это корни (комплексные или действительные).

    Отсюда видно, что само разложение сводится к решению квадратного уравнения впоследствии.

    Пример 1

    Произвести разложение квадратного трехчлена на множители.

    Решение

    Необходимо найти корни уравнения 4×2-5x+1=0 . Для этого необходимо найти значение дискриминанта по формуле, тогда получим D=(-5)2-4·4·1=9 . Отсюда имеем, что

    x1=5-92·4=14×2=5+92·4=1

    Отсюда получаем, что 4×2-5x+1=4x-14x-1.

    Для выполнения проверки нужно раскрыть скобки. Тогда получим выражение вида:

    4x-14x-1=4×2-x-14x+14=4×2-5x+1

    После проверки приходим к исходному выражению. То есть можно сделать вывод, что разложение выполнено верно.

    Пример 2

    Произвести разложение на множители квадратный трехчлен вида 3×2-7x-11.

    Решение

    Получим, что необходимо вычислить получившееся квадратное уравнение вида 3×2-7x-11=0.

    Чтобы найти корни, надо определить значение дискриминанта. Получим, что

    3×2-7x-11=0D=(-7)2-4·3·(-11)=181×1=7+D2·3=7+1816×2=7-D2·3=7-1816

    Отсюда получаем, что 3×2-7x-11=3x-7+1816x-7-1816 .

    Пример 3

    Произвести разложение многочлена 2×2+1  на множители.

    Решение

    Теперь нужно решить квадратное уравнение 2×2+1=0 и найти его корни. Получим, что

    2×2+1=0x2=-12×1=-12=12·ix2=-12=-12·i

    Эти корни называют комплексно сопряженными, значит само разложение можно изобразить как 2×2+1=2x-12·ix+12·i .

    Пример 4

    Произвести разложение квадратного трехчлена x2+13x+1.

    Решение

    Для начала необходимо решить квадратное уравнение вида x2+13x+1=0  и найти его корни.

    x2+13x+1=0D=132-4·1·1=-359×1=-13+D2·1=-13+353·i2=-1+35·i6=-16+356·ix2=-13-D2·1=-13-353·i2=-1-35·i6=-16-356·i

    Получив корни, запишем

    x2+13x+1=x—16+356·ix—16-356·i==x+16-356·ix+16+356·i

    Замечание

    Если значение дискриминанта отрицательное, то многочлены останутся многочленами второго порядка. Отсюда следует, что раскладывать их не будем на линейные множители.

    Способы разложения на множители многочлена степени выше второй

    При разложении предполагается универсальный метод. Большинство всех случаев основано на следствии из теоремы Безу. Для этого необходимо подбирать значение корня x1 и понизить его степень при помощи деления на многочлена на 1 делением на (x-x1) . Полученный многочлен нуждается  в нахождении корня x2 , причем процесс поиска цикличен до тех пор, пока не получим полное разложение.

    Если корень не нашли, тогда применяются другие способы разложения на множители: группировка, дополнительные слагаемые. Данная тема полагает решение уравнений с высшими степенями  и целыми коэффициентами.

    Вынесение общего множителя за скобки

    Рассмотрим случай, когда свободный член равняется нулю, тогда вид многочлена становится как Pn(x)=anxn+an-1xn-1+…+a1x.

    Видно, что корень такого многочлена будет равняться x1=0 , тогда можно представить многочлен в виде выражения Pn(x)=anxn+an-1xn-1+…+a1x==x(anxn-1+an-1xn-2+…+a1)

    Данный способ считается вынесением общего множителя за скобки.

    Пример 5

    Выполнить разложение многочлена третьей степени 4×3+8×2-x  на множители.

    Решение

    Видим, что x1=0  — это корень заданного многочлена, тогда можно произвести вынесение х за скобки всего выражения. Получаем:

    4×3+8×2-x=x(4×2+8x-1)

    Переходим к нахождению корней квадратного трехчлена 4×2+8x-1 .  Найдем дискриминант и корни:

    D=82-4·4·(-1)=80×1=-8+D2·4=-1+52×2=-8-D2·4=-1-52

    Тогда следует, что

    4×3+8×2-x=x4x2+8x-1==4xx—1+52x—1-52==4xx+1-52x+1+52

    Разложение на множители многочлена с рациональными корнями

    Для начала примем за рассмотрение способ разложения, содержащий целые коэффициенты вида Pn(x)=xn+an-1xn-1+…+a1x+a0 , где коэффициента при старшей степени равняется 1.

    Когда многочлен имеет целые корни, тогда их считают делителями свободного члена.

    Пример 6

    Произвести разложение выражения f(x)=x4+3×3-x2-9x-18 .

    Решение

    Рассмотрим, имеются ли целые корни. Необходимо выписать делители числа -18. Получим, что ±1,±2,±3,±6,±9,±18. Отсюда следует, что данный многочлен имеет целые корни. Можно провести проверку по схеме Горнера. Она очень удобная и позволяет быстро получить  коэффициенты разложения многочлена:

    xi Коэффициенты многочленов
      1 3 -1 -9 -18
    1 1 3+1·1=4 -1+4·1=3 -9+3·1=-6 -18+(-6)·1=-24
    -1 1 3+1·(-1)=2 -1+2·(-1)=-3 -9+(-3)·(-1)=-6 -18+(-6)·(-1)=-12
    2 1 3+1·2=5 -1+5·2=9 -9+9·2=9 -18+9·2=0
    2 1 5+1·2=7 9+7·2=23 9+23·2=55  
    -2 1 5+1·(-2)=3 9+3·(-2)=3 9+3·(-2)=3  
    3 1 5+1·3=8 9+8·3=33 9+33·3=108  
    -3 1 5+1·(-3)=2 9+2·(-3)=3 9+3·(-3)=0  

    Отсюда следует, что х=2 и х=-3 – это корни исходного многочлена, который можно представить как произведение вида:

    f(x)=x4+3×3-x2-9x-18=(x-2)(x3+5×2+9x+9)==(x-2)(x+3)(x2+2x+3)

    Переходим к разложению квадратного трехчлена вида x2+2x+3.

    Так как дискриминант получаем отрицательный, значит, действительных корней нет.

    Ответ: f(x)=x4+3×3-x2-9x-18=(x-2)(x+3)(x2+2x+3)

    Замечание

    Допускается использование подбором корня и деление многочлена на многочлен вместо схемы Горнера. Перейдем к рассмотрению разложения многочлена, содержащим целые коэффициенты вида Pn(x)=xn+an-1xn-1+…+a1x+a0, старший из которых на равняется единице.

    Этот случай имеет место быть для дробно-рациональных дробей.

    Пример 7

    Произвести разложение на множители f(x)=2×3+19×2+41x+15.

    Решение

    Необходимо выполнить замену переменной y=2x, следует переходить  к многочлену с коэффициентами равными 1 при старшей степени. Необходимо начать с умножения выражения на 4. Получаем, что

    4f(x)=23·x3+19·22·x2+82·2·x+60==y3+19y2+82y+60=g(y)

    Когда получившаяся функция  вида g(y)=y3+19y2+82y+60 имеет целые корни, тогда их нахождение среди делителей свободного члена. Запись примет вид:

    ±1,±2,±3,±4,±5,±6,±10,±12,±15,±20,±30,±60

    Перейдем  к вычислению функции g(y) в этих точка для того, чтобы получить в результате ноль. Получаем, что

    g(1)=13+19·12+82·1+60=162g(-1)=(-1)3+19·(-1)2+82·(-1)+60=-4g(2)=23+19·22+82·2+60=308g(-2)=(-2)3+19·(-2)2+82·(-2)+60=-36g(3)=33+19·32+82·3+60=504g(-3)=(-3)3+19·(-3)2+82·(-3)+60=-42g(4)=43+19·42+82·4+60=756g(-4)=(-4)3+19·(-4)2+82·(-4)+60=-28g(5)=53+19·52+82·5+60=1070g(-5)=(-5)3+19·(-5)2+82·(-5)+60

    Получаем, что у=-5 – это корень уравнения вида y3+19y2+82y+60, значит, x=y2=-52 — это корень исходной функции.

    Пример 8

    Необходимо произвести деление столбиком 2×3+19×2+41x+15  на x+52 . 

    Решение

    Запишем и получим:

    Значит,

    2×3+19×2+41x+15=x+52(2×2+14x+6)==2x+52(x2+7x+3)

    Проверка делителей займет много времени, поэтому выгодней предпринять разложение на множители полученного квадратного трехчлена вида x2+7x+3. Приравниванием к нулю и находим дискриминант.

    x2+7x+3=0D=72-4·1·3=37×1=-7+372×2=-7-372⇒x2+7x+3=x+72-372x+72+372

    Отсюда следует, что

    2×3+19×2+41x+15=2x+52×2+7x+3==2x+52x+72-372x+72+372

    Искусственные приемы при  разложении многочлена на множители

    Рациональные корни не присущи всем многочленам. Для этого необходимо пользоваться специальными способами для нахождения множителей. Но не все многочлены можно разложить или представить в виде произведения.

    Способ группировки

    Бывают случаи, когда можно сгруппировывать слагаемые многочлена для нахождения общего множителя и вынесения его за скобки.

    Пример 9

    Произвести разложение многочлена x4+4×3-x2-8x-2 на множители.

    Решение

    Потому как коэффициенты – целые числа, тогда корни предположительно тоже могут быть целыми. Для проверки возьмем значения 1, -1, 2 и -2 для того, чтобы вычислить значение многочлена в этих точках. Получаем, что

    14+4·13-12-8·1-2=-6≠0(-1)4+4·(-1)3-(-1)2-8·(-1)-2=2≠024+4·23-22-8·2-2=26≠0(-2)4+4·(-2)3-(-2)2-8·(-2)-2=-6≠0

    Отсюда видно, что корней нет, необходимо использовать другой способ разложения и решения.

    Необходимо провести группировку:

    x4+4×3-x2-8x-2=x4+4×3-2×2+x2-8x-2==(x4-2×2)+(4×3-8x)+x2-2==x2(x2-2)+4x(x2-2)+x2-2==(x2-2)(x2+4x+1)

    После группировки исходного многочлена необходимо представить его как произведение двух квадратных трехчленов. Для этого нам понадобится произвести разложение на множители. получаем, что

    x2-2=0x2=2×1=2×2=-2⇒x2-2=x-2x+2×2+4x+1=0D=42-4·1·1=12×1=-4-D2·1=-2-3×2=-4-D2·1=-2-3⇒x2+4x+1=x+2-3x+2+3

    Значит:

    x4+4×3-x2-8x-2=x2-2×2+4x+1==x-2x+2x+2-3x+2+3

    Замечание

    Простота группировки не говорит о том, что выбрать слагаемы достаточно легко. Определенного способа решения не существует, поэтому необходимо пользоваться специальными теоремами и правилами.

    Пример 10

    Произвести разложение на множители многочлен x4+3×3-x2-4x+2 .

    Решение

    Заданный многочлен не имеет целых корней. Следует произвести группировку слагаемых. Получаем, что

    x4+3×3-x2-4x+2==(x4+x3)+(2×3+2×2)+(-2×2-2x)-x2-2x+2==x2(x2+x)+2x(x2+x)-2(x2+x)-(x2+2x-2)==(x2+x)(x2+2x-2)-(x2+2x-2)=(x2+x-1)(x2+2x-2)

    После разложения на множители получим, что

    x4+3×3-x2-4x+2=x2+x-1×2+2x-2==x+1+3x+1-3x+12+52x+12-52

    Использование формул сокращенного умножения и бинома Ньютона для разложения многочлена на множители

    Внешний вид зачастую не всегда дает понять, каким способом необходимо воспользоваться при разложении. После того, как были произведены преобразования, можно выстроить строчку, состоящую из треугольника Паскаля, иначе их называют биномом Ньютона.

    Пример 11

    Произвести разложение многочлена x4+4×3+6×2+4x-2  на множители.

    Решение

    Необходимо выполнить преобразование выражения к виду

    x4+4×3+6×2+4x-2=x4+4×3+6×2+4x+1-3

    На последовательность коэффициентов суммы в скобках указывает выражение x+14.

    Значит, имеем x4+4×3+6×2+4x-2=x4+4×3+6×2+4x+1-3=x+14-3.

    После применения разности квадратов, получим

    x4+4×3+6×2+4x-2=x4+4×3+6×2+4x+1-3=x+14-3==x+14-3=x+12-3x+12+3

    Рассмотрим выражение, которое находится во второй скобке. Понятно, что там коней нет, поэтому следует применить формулу разности квадратов еще раз. Получаем выражение вида

    x4+4×3+6×2+4x-2=x4+4×3+6×2+4x+1-3=x+14-3==x+14-3=x+12-3x+12+3==x+1-34x+1+34×2+2x+1+3

    Пример 12

    Произвести разложение на множители x3+6×2+12x+6.

    Решение

    Займемся преобразованием выражения. Получаем, что

    x3+6×2+12x+6=x3+3·2·x2+3·22·x+23-2=(x+2)3-2

    Необходимо применить формулу сокращенного умножения разности кубов. Получаем:

    x3+6×2+12x+6==(x+2)3-2==x+2-23x+22+23x+2+43==x+2-23×2+x2+23+4+223+43

    Способ замены переменной при разложении многочлена на множители

    При замене переменной производится понижение степени и разложение многочлена на множители.

    Пример 13

    Произвести разложение на множители многочлена вида x6+5×3+6.

    Решение

    По условию видно, что необходимо произвести замену y=x3 . Получаем:

    x6+5×3+6=y=x3=y2+5y+6

    Корни полученного квадратного уравнения равны y=-2 и y=-3, тогда

    x6+5×3+6=y=x3=y2+5y+6==y+2y+3=x3+2×3+3

    Необходимо применить формулу сокращенного умножения суммы кубов. Получим выражения вида:

    x6+5×3+6=y=x3=y2+5y+6==y+2y+3=x3+2×3+3==x+23×2-23x+43x+33×2-33x+93

    То есть получили искомое разложение.

    Рассмотренные выше случаи помогут в рассмотрении  и разложении многочлена на множители разными способами.

    Факторинг — Математика GCSE — шаги, примеры и рабочий лист

    Вот все, что вам нужно знать о факторинге для математики GCSE (Edexcel, AQA и OCR). Вы изучите основы факторизации выражений и факторизации квадратичных чисел, включая факторизацию в одинарные и двойные скобки.

    Обратите внимание на рабочие листы факторинга и экзаменационные вопросы в конце.

    Что такое факторизация

    Факторизация — это процесс, обратный раскрытию скобок. Чтобы полностью разложить выражение на множители, нужно заключить его в скобки, вынеся старшие общие множители.

    Простейший способ разложения на множители:

    • Найдите наибольший общий делитель каждого члена выражения.
    • Запишите наибольший общий делитель (HCF) перед любыми скобками
    • Заполните каждый термин в скобках путем умножения.

    Однако существуют разные способы разложения на множители различных типов алгебраических выражений; мы узнаем о них всех здесь.

    Что такое факторинг?

    Таблица по факторингу (смешанная)

    Получите бесплатную таблицу по факторингу, содержащую более 20 вопросов и ответов. Включает рассуждения и прикладные вопросы.

    СКАЧАТЬ БЕСПЛАТНО

    Икс

    Рабочий лист по факторингу (смешанный)

    Получите бесплатный рабочий лист по факторингу, содержащий более 20 вопросов и ответов. Включает рассуждения и прикладные вопросы.

    СКАЧАТЬ БЕСПЛАТНО

    Как факторизовать выражения

    Для факторизации алгебраических выражений существует три основных метода. Когда вы факторизуете квадратные числа, вы обычно используете метод двойных скобок или разности двух квадратов.

    1. Разложение одинарных скобок на множители

    Пример разложения на множители алгебраического выражения:

    Помните: 3x+6 называется биномом, потому что это выражение с двумя членами

    2. Факт двойные упорные скобы

    а) При разложении квадратных выражений вида x 2 + bx + c

    b) При разложении квадратных выражений вида ax 2 + bx + c

    Помните:
    Выражения с тремя членами типа x 2 + 6x + 5 и 2x 2 + 5x + 3 известны как трехчлены.

    3. Разность двух квадратов


    Используя разность двух квадратов:

    Объясните, как разложить выражения на множители

    Методы факторизации

    Каждый метод факторизации или факторизации выражений приводится ниже. Для получения подробных примеров, практических вопросов и рабочих листов по каждому из них следуйте ссылкам на пошаговые руководства.

    1. Факторизация одинарных скобок

    Пример факторизации с использованием одинарных скобок

    Чтобы полностью разложить на множители:

    \[\color{#00BC89}3x + \color{#7C4DFF}6\]

    1. Найдите наибольший общий множитель ( HCF) чисел 3 (коэффициент при x) и 6 (константа).

    Факторы 3:
    1, 3

    Факторы 6:
    1, 6
    2, 3


    Совет:

    Запись пар множителей облегчает перечисление всех факторов

    Наибольший общий делитель (HCF) чисел 3x и 6 равен 3

    2 Запишите наибольший общий делитель (HCF) перед одиночной скобкой.

    \[\color{#FF9100}3(\quad+\quad)\]

    3 Заполните каждый термин в скобках путем умножения.

    На что мне нужно умножить 3, чтобы получить 3x?

    \[\color{#FF9100}3 \times \color{#62F030}x = \color{#00BC89}3 x\]

    На что мне нужно умножить 3, чтобы получить 6?

    \[\color{#FF9100}3 \times \color{#92009E}2 = \color{#7C4DFF}6\]

    \[\color{#FF9100}3(\color{#62F030}x + \color{#92009E}2)\]

    Мы можем проверить ответь умножением скобки!

    \[3(x+2)=3 x+6\]

    Пошаговое руководство: Разложение одинарных скобок на множители

    Пример факторизации квадратного выражения в форме x 2 + bx + c

    Чтобы полностью разложить на множители: 92 + \color{#00bc89}6x + \color{#7C4DFF}5\]

    1. Выпишите пары множителей последнего числа (5)

    Делители 5:
    1, 5

    2 Найдите пару делителей, которые + дают среднее число (6) и ✕ дают последнее число (5).

      1  +  5  =  6  1  5  =  5 

    3 Запишите две скобки и поставьте переменную в начале каждой.

    \[(х\qquad)(х\qquad)\] 92 + \color{#00bc89}5x \color{#7C4DFF}{+3}\]

    1. Умножьте конечные числа (2 и 3), затем запишите пары множителей этого нового числа в порядке

    Делители 6:
    1, 6
    2, 3

    2 Нам нужна пара множителей, которые + дают среднее число (5) и ✕ дают это новое число (6)

    2 + 3 = 5 ✔

    2 ✕ 3 = 6 ✔

    3 Перепишите исходное выражение, на этот раз разделив средний член на два множителя, которые мы нашли на шаге 2. 9{2}\color{#FF9100}{+2 x+3 x}+3\]

    4 Разделите уравнение пополам и полностью разложите каждую половину на множители.

    \[\color{#398CDA}{2 x}\color{#62F030}{(x+1)}\color{#398CDA}{+3}\color{#62F030}{(x+1)} \]

    5 Разложите все выражение на множители, вынеся все, что находится в скобках, на передний план и запишите два других члена в другой скобке.

    \[(\color{#398CDA}{2x+ 3})\color{#62F030}{(x + 1)}\]

    Пошаговое руководство: Факторизация квадратичных уравнений 92}}=\color{#FE47EC}{2 x}\]

    \[(\color{#FE47EC}{2x}\qquad)(\color{#FE47EC}{2x}\qquad)\]

    3 Извлеките корень из последнего члена и запишите его справа от обеих скобок.

    \[\sqrt{\color{#7C4DFF}9}=\pm\color{#7C4DFF}3\]

    \[(\color{#FE47EC}{2x}\quad\color{#7C4DFF}3 )(\color{#FE47EC}{2x}\quad\color{#7C4DFF}3)\]

    4 Поставьте + в середине одной скобки и – в середине другой (порядок не имеет значения) .

    \[(\color{#FE47EC}{2x}+\color{#7C4DFF}3)(\color{#FE47EC}{2x}-\color{#7C4DFF}3)\] 9{2}-2х-3)

    Чтобы разложить на множители квадратное выражение, мы ищем числа, которые умножаются на -3 и в сумме дают -2. Рассматривая пары факторов, мы приходим к выводу, что нам нужно использовать +1 и -3.

    2(х+1)(х-3)

    (х+3)(х+3)

    (х+1)(х-9)

    (х+3)(х-3)

    (х+1)(х-3)

    Это частный случай (разность двух квадратов), а это значит, что мы можем взять квадратные корни из коэффициента при х и постоянного члена, затем записать одну скобку со знаком +, а другую скобку со знаком –.

    (3x+4)(3x-4)

    (3x-4)(3x-4)

    (9x+16)(x-1)

    (3x+1)(3x-16)

    Это частный случай (разность двух квадратов), а это значит, что мы можем взять квадратные корни из коэффициента при х и постоянного члена, затем записать одну скобку со знаком +, а другую скобку со знаком –.

    Факторинг вопросов GCSE (смешанный)

    1. Разложить на множители: 9x − 18

    Показать ответ

    9(x − 2)

    (1 балл)

    2. Разложить на множители полностью: 16x 2 + 20xy

    Показать ответ

    4x(4x + 5y)

    (2 балла)

    3. Разложить полностью: 3y 2 − 4y − 4

    Показать ответ

    (3 года + 2)( у — 2)

    (2 балла)

    Учебный контрольный список

    Теперь вы научились:

    • Манипулировать алгебраическими выражениями, вынося общие множители и разлагая их в одну скобку.
    • Факторизация квадратичных выражений вида x 2 + bx + c
    • Разложить на множители квадратные выражения в виде разности двух квадратов.
    • Факторизация квадратичных выражений вида ax 2 + bx + c (H)

    Все еще зависает?

    Подготовьте своих учеников KS4 к успешной сдаче выпускных экзаменов по математике с помощью программы Third Space Learning. Еженедельные онлайн-уроки повторения GCSE по математике, которые проводят опытные преподаватели математики.

    Узнайте больше о нашей программе повторения GCSE по математике.

    Факторинг в алгебре

    Факторы

    Числа имеют множители:

    И выражения (например, x 2 +4x+3 ) также имеют множители:

    Факторинг

    Факторинг (называемый « Факторинг » в Великобритании) — это процесс нахождения факторов :

    Факторинг: поиск того, что нужно перемножить, чтобы получить выражение.

    Это похоже на «разбиение» выражения на произведение более простых выражений.

    Пример: коэффициент 2y+6

    И 2y, и 6 имеют общий делитель 2:

    • 2y равно 2×y
    • 6 равно 2×3

    Итак, мы можем разложить все выражение на:

    2y+6 = 2(y+3)

    Таким образом, 2y+6 было «разложено в» 2 и y+3

    Факторинг также противоположен расширению:

    Общий коэффициент

    В предыдущем примере мы видели, что 2y и 6 имеют общий делитель 9. 0110 2

    Но для правильной работы нам нужен наибольший общий делитель , включая любые переменные

    Пример: множитель 3y

    2 +12y

    Во-первых, 3 и 12 имеют общий делитель 3.

    Таким образом, мы могли бы иметь: +4г)

    Но мы можем лучше!

    3y 2 и 12y также имеют общую переменную y.

    Вместе это составляет 3 года:

    • 3 года 2 это 3г × г
    • 12 лет — это 3 года × 4

     

    Таким образом, мы можем разложить все выражение на:

    3y 2 +12y = 3y(y+4)

     

    Проверить: 3y(y+4) = 3 у × у + 3у × 4 = 3 года 2 +12 лет

    Более сложный факторинг

    Факторинг может быть сложным!

    До сих пор примеры были простыми, но разложение на множители может быть очень сложным.

    Потому что мы должны цифру то, что было умножено на , чтобы получить выражение, которое нам дано!

     


    Это все равно, что пытаться выяснить, какие ингредиенты
    вошли в торт, чтобы сделать его таким вкусным.
    Это может быть трудно понять!

    Опыт помогает

    Чем больше опыта, тем проще факторинг.

    Пример: Фактор

    4x 2 − 9

    Хммм… кажется, нет никаких общих факторов.

    Но знание специальных биномиальных произведений дает нам подсказку, называемую «разность квадратов» :

    Потому что 4x 2 равно (2x) 2 и 9 9011 2 равно (3) 2 ,

    Итак, мы имеем:

    4x 2 − 9 = (2x) 2 − (3) 2

    И это можно получить по формуле разности квадратов:

    (a+b)(a−b) = a 2 − b 2

    Где a равно 2x, а b равно 3.

    Попробуем сделать так:

    (2x+3)(2x−3) = (2x) 2 − (3) 2 = 4x 2 − 9

    Да!

     

    Итак, множители 4x 2 − 9 равны (2x+3) и (2x−3) :

    Ответ: 4x 90 060 2 — 9 = (2x+3)( 2x−3)

    Как этому научиться? Получив много практики и зная «Идентичности»!

     

    Запомнить эти личности

    Вот список общих «Идентификаций» (включая «разность квадратов» , использованную выше).

    Их стоит запомнить, так как они могут упростить факторинг.

    а 2 − б 2  =  (а+б)(а-б)
    а 2 + 2аб + б 2  =  (а+б)(а+б)
    а 2 − 2аб + б 2  =  (а-б) (а-б)
    а 3 + б 3  =  (а+б)(а 2 −аб+б 2 )
    а 3 − б 3  =  (а-б)(а 2 +аб+б 2 )
    а 3 +3а 2 б+3аб 2 3  =  (а+б) 3
    а 3 −3а 2 б+3аб 2 −b 3  =  (а-б) 3

    Таких много, но эти самые полезные.

    Совет

    Обычно лучше использовать факторизованную форму.

    При попытке факторинга выполните следующие действия:

    • «Вынести за скобки» любые общие термины
    • Посмотрите, подходит ли оно к какой-либо из идентификаций, а также к тому, что вы знаете
    • Продолжайте, пока не перестанете множить

    Существуют также системы компьютерной алгебры (называемые «CAS»), такие как Axiom, Derive, Macsyma, Maple, Mathematica, MuPAD, Reduce и другие, которые могут выполнять факторинг.

    Другие примеры

    Опыт помогает, поэтому вот еще несколько примеров, которые помогут вам на этом пути:

    Пример: w

    4 − 16

    Показатель степени 4? Может быть, мы могли бы попробовать показатель степени 2:

    w 4 − 16 = (w 2 ) 2 − 4 2

    Да, это разность квадратов

    90 002 ш 4 − 16 = (w 2 + 4)(w 2 − 4)

    И «(w 2 − 4)» — еще одна разность квадратов

    w 4 − 16 = (w 2 + 4)(w+ 2)(w− 2)

    Это все, что я могу сделать (если я не использую мнимые числа)

    Пример: 3u

    4 − 24uv 3

    Удалить общий делитель «3u»:

    3u 4 − 24uv 3 = 3u(u 9006 0 3 − 8v 3 )

    Тогда разность кубов:

    3u 4 − 24uv 3 = 3u(u 3 − (2v) 3 )

    = 3u(u−2v)(u 9006 0 2 +2ув+4в 2 )

    Это все, что я могу сделать.

    График у 1 корень из х: График y = f(x) = 1/sqrt(x) (1 делить на квадратный корень из (х)) постройте график функции и изобразите его. Исследуйте данную функцию. [Есть ответ!]

    Функция корень из х. Ее свойства и график

    Похожие презентации:

    Функция у х, её свойства и график

    Функция у=n квадратный корень из х, их свойства и графики

    Функция у=√х, её свойства и график

    Свойства и график функции у = sin x

    Функции и их свойства

    Функция у=kx², её свойства и график

    Функция у=кх², её свойства и график

    Логарифмическая функция, ее свойства и график

    Функция у=х в степени n , график и свойства

    Функции. Их свойства и графики

    Функция
    у х,
    её свойства и график.
    Решить уравнения:
    a )2 x 8 10;
    1
    б)
    x 9 6;
    3
    в) x x 2
    1)Вычислите:
    11
    1
    0,36 ; 1 ;0,2 400 ; 81;
    25
    3
    0,49 0,09 ; 2 1; 5 12 .
    3
    2)Решите уравнение:
    x 4; x 4
    2
    x 1; x 1
    2
    2
    2
    Решите уравнение
    1
    x 5
    3
    1
    x 52
    3
    1
    x 25
    3
    x 75
    Ответ : 75
    Решите уравнение:
    а )( x 3)( x 1) 0
    б )( x 2)( x 1) 0
    Вычислите, укажите правильный
    ответ
    64
    36
    63
    7
    1,2
    2
    3
    16
    14
    8
    5
    12
    10
    3,2
    Вычислите, укажите правильный
    ответ
    169 16
    63
    7
    1,2
    2
    3
    16
    150
    8
    5
    12
    10
    3,2
    Какое целое число заключено между
    45 и
    54
    63
    7
    1,2
    2
    180
    16
    150
    8
    5
    12
    10
    3,2
    Что больше?
    π; 5 ;3,2
    63
    100
    1,2
    2
    180
    16
    150
    8
    5
    12
    10
    3,2
    Найти наибольшее значение
    функции
    y= x на отрезке [1;25]
    63
    100
    1,2
    2
    180
    16
    150
    8
    5
    12
    10
    270
    Решить уравнение:
    x =4
    63
    100
    1,2
    2
    180
    16
    150
    8
    260
    12
    10
    270
    Найти наибольший корень
    уравнения:
    x 4
    2
    63
    100
    1,2
    2
    180
    280
    150
    8
    260
    12
    10
    270
    Вычислите, укажите правильный
    ответ
    49
    81
    63
    100
    1,2
    290
    180
    280
    150
    8
    260
    12
    10
    270
    Вычислите, укажите правильный
    ответ
    81 1
    300
    100
    1,2
    290
    180
    280
    150
    8
    260
    12
    10
    270
    Вычислите, укажите правильный
    ответ
    11
    1
    25
    300
    100
    1,2
    290
    180
    280
    150
    8
    260
    12
    310
    270
    Найти сторону квадрата, если его
    площадь равна
    64 см
    2
    300
    100
    320
    290
    180
    280
    150
    8
    260
    12
    310
    270
    Найти периметр квадрата, если его
    площадь равна
    2
    см
    9
    300
    100
    320
    290
    180
    280
    150
    330
    260
    12
    310
    270

    18.

    Арифметический квадратный кореньАРИФМЕТИЧЕСКИЙ
    КВАДРАТНЫЙ КОРЕНЬ
    выражение
    -3 49
    уравнение
    x 8
    функция
    ?
    у х
    х≥0
    Х
    0
    У
    0
    1 2,25 4 6,25 9
    1 1,5 2 2,5 3
    у
    3
    2
    1
    0 1 2 3 4 5 6 7 8 9
    х
    Свойства функции
    1.Область
    1. D ( у ) 0 ;
    определения
    у
    2.
    2.Область
    значений
    3. у=0, если х= 0 4
    у>0, если
    3
    у=√х:
    E ( у ) 0;
    х 0;
    1
    х
    4. Функция
    -1 0 1 2 3 4 5 6 7 8 9 10 11
    возрастает
    при х 0 ;
    ограничена снизу, но не
    5. Функция
    Ограниченность
    ограничена сверху.
    6. унаим.= 0
    Непрерывна.
    7. Непрерывность
    7.
    унаиб.= НЕТ
    у х
    х≥0
    Х
    0
    У
    0
    1 2,25 4 6,25 9
    -1 -1,5 -2 -2,5 -3
    у
    х
    -1
    -2
    -3
    -4
    0 1 2 3 4 5 6 7 8 9 10 11
    Свойства функции
    1.Область
    1. D ( у ) 0 ;
    определения
    2.
    2.Область
    значений
    3. у=0, если х= 0
    E ( у ) ;0
    у<0, если
    х 0;
    4. Функция
    убывает
    при х 0 ;
    -1
    -2
    -3
    -4
    у=-√х:
    у
    х
    0 1 2 3 4 5 6 7 8 9 10 11
    ограничена сверху, и не
    5. Функция
    Ограниченность
    ограничена снизу.
    6. унаим.= НЕТ
    Непрерывна.
    7. Непрерывность
    7.
    унаиб.= 0
    Постройте график
    функции:
    у
    y х 3 4
    система
    координат:
    х= 3
    у= 4
    -2 -1
    2. Привязываем к
    ней график функции
    х
    y х 3 4
    7
    6
    5
    4
    3
    2
    1
    1.Вспомогательная
    y
    х=3
    -1
    -2
    у=4
    х
    01 2 3 4 5 6 7 8 9
    Х
    0
    1
    У
    0
    1
    4
    2
    Найдите наименьшее и наибольшее значения
    у х
    функции
    на отрезке от 0 до 4.
    у
    4
    3
    2
    1
    у х
    х
    -1 0 1 2 3 4 5 6 7 8 9 10 11
    Унаим.=0
    Унаиб.=2
    Найдите наименьшее и наибольшее значения
    функции у х 2 на отрезке от 3 до 11.
    у
    х=2
    4
    3
    у х 2
    1
    х
    -1 0 1 2 3 4 5 6 7 8 9 10 11
    Унаим.=1
    Унаиб. =3
    Решить графически уравнение:
    √х=х-6
    1
    4
    3
    2
    Построим в одной системе
    координат графики функций:
    у=√х
    у=х-6
    Х 0 6
    У -6 0
    2
    3
    у х
    1
    -1
    -2
    -3
    -4
    -5
    -6
    Х 0 1 4 9
    У 0 1 2 3
    у
    х
    0 1 2 3 4 5 6 7 8 9
    у=х-6
    Найдём абсциссы точек
    пересечения графиков
    ОТВЕТ:
    9
    Решить графически систему уравнений:
    у
    9
    8
    7
    Построим в одной системе
    1 координат графики функций: 6
    5
    у=(х-3)²
    4
    В.С.К. х=3, у=0
    3
    у=х²
    2
    Х 0 ±1 ±2±3
    1
    у=(х-3)²
    у=√х-3
    У 0 1 4 9
    у=√х-3
    В.С.К. х=3, у=0
    0
    4
    Х
    1
    у=√х
    У 0 1 2
    -1
    х=3
    у=(х-3)²
    у=√х-3
    (4;1)
    у=0
    01 2 3 4 5 6 7 8 9 х
    (3;0)
    Найдём координаты точек
    пересечения графиков
    2
    3 ОТВЕТ
    (3;0) , (4;1)

    English     Русский Правила

    График функции 1 корень. Степенная функция и корни

    Квадратный корень как элементарная функция.

    Квадратный корень — это элементарная функция и частный случай степенной функции при . Арифметический квадратный корень является гладким при , а нуле он непрерывен справа, но не дифференцируется.

    Как функция комплексный переменный корень — двузначная функция, у которой листы сходятся в нуле.

    Построение графика функции квадратного корня.

    1. Заполняем таблицу данных:

    2. Наносим точки, которые мы получили на координатную плоскость.

    3. Соединяем эти точки и получаем график функции квадратного корня:

    Преобразования графика функции квадратного корня.

    Определим, какие преобразования функции необходимо сделать для того, чтобы построить графики функций. Определим виды преобразований.

    Вид преобразования

    Преобразование

    Перенос функции по оси OY на 4 ед. вверх.

    внутреннее

    Перенос функции по оси OX на 1 ед. вправо.

    внутреннее

    График приближается к оси OY в 3 раза и сжимается по оси .

    График отдаляется от оси OX OY .

    внутреннее

    График отдаляется от оси OY в 2 раза и растягивается по оси .

    Зачастую преобразования функций оказываются комбинированными.

    Например , нужно построить график функции . Это график квадратного корня , который нужно перенести на одну единицу вниз по оси OY и на единицу вправо по оси ОХ и одновременно растянув в 3 раза его по оси OY .

    Бывает непосредственно перед построением графика функции, нужны предварительные тождественные преобразования либо упрощения функций.

    Основные цели:

    1) сформировать представление о целесообразности обобщённого исследования зависимостей реальных величин на примере величин, связанных отношением у=

    2) формировать способность к построению графика у= и его свойства;

    3) повторить и закрепить приёмы устных и письменных вычислений, возведение в квадрат, извлечение квадратного корня.

    Оборудование, демонстрационный материал: раздаточный материал.

    1. Алгоритм:

    2. Образец для выполнения задания в группах:

    3. Образец для самопроверки самостоятельной работы:

    4. Карточка для этапа рефлексии:

    1) Я понял, как построить график функции у=.

    2) Я могу по графику перечислить его свойства.

    3) Я не допустил ошибок в самостоятельной работе.

    4) Я допустил ошибки в самостоятельной работе (перечислить эти ошибки и указать их причину).

    Ход урока

    1. Самоопределение к учебной деятельности

    Цель этапа:

    1) включить учащихся в учебную деятельность;

    2) определить содержательные рамки урока: продолжаем работать с действительными числами.

    Организация учебного процесса на этапе 1:

    – Что мы изучали на прошлом уроке? (Мы изучали множество действительных чисел, действия с ними, построили алгоритм для описания свойств функции, повторяли функции изученные в 7 классе).

    – Сегодня мы продолжим работать с множеством действительных чисел, функцией.

    2. Актуализация знаний и фиксация затруднений в деятельности

    Цель этапа:

    1) актуализировать учебное содержание, необходимое и достаточное для восприятия нового материала: функция, независимая переменная, зависимая переменна, графики

    y = kx + m, y = kx, y =c, y =x 2 , y = — x 2 ,

    2) актуализировать мыслительные операции, необходимые и достаточные для восприятия нового материала: сравнение, анализ, обобщение;

    3) зафиксировать все повторяемые понятия и алгоритмы в виде схем и символов;

    4) зафиксировать индивидуальное затруднение в деятельности, демонстрирующее на личностно значимом уровне недостаточность имеющихся знаний.

    Организация учебного процесса на этапе 2:

    1. Давайте вспомним как можно задать зависимости между величинами? (С помощью текста, формулы, таблицы, графика)

    2. Что называется функцией? (Зависимость между двумя величинами, где каждому значению одной переменной соответствует единственное значение другой переменной y = f(x)).

    Как называется х? (Независимая переменная — аргумент)

    Как называется у? (Зависимая переменная).

    3. В 7- м классе мы изучили функции? (y = kx + m, y = kx, y =c, y =x 2 , y = — x 2 , ).

    Индивидуальное задание:

    Что является графиком функций y = kx + m, y =x 2 , y = ?

    3. Выявление причин затруднений и постановка цели деятельности

    Цель этапа:

    1) организовать коммуникативное взаимодействие, в ходе которого выявляется и фиксируется отличительное свойство задания, вызвавшего затруднение в учебной деятельности;

    2) согласовать цель и тему урока.

    Организация учебного процесса на этапе 3:

    – Что особенного в этом задании? (Зависимость задана формулой y = с которой мы еще не встречались).

    – Какая цель урока? (Познакомиться с функцией y = , ее свойствами и графиком. Функцией в таблице определять вид зависимости, строить формулу и график.)

    – Можно сформулировать тему урока? (Функция у=, ее свойства и график).

    – Запишите тему в тетради.

    4. Построение проекта выхода из затруднения

    Цель этапа:

    1) организовать коммуникативное взаимодействие для построения нового способа действия, устраняющего причину выявленного затруднения;

    2) зафиксировать новый способ действия в знаковой, вербальной форме и с помощью эталона.

    Организация учебного процесса на этапе 4:

    Работу на этапе можно организовать по группам, предложив группам построить график y = , затем проанализировать получившиеся результаты. Также группам можно предложить по алгоритму описать свойства данной функции.

    5. Первичное закрепление во внешней речи

    Цель этапа: зафиксировать изученное учебное содержание во внешней речи.

    Организация учебного процесса на этапе 5:

    Постройте график у= — и опишите его свойства.

    Свойства у= — .

    1.Область определения функции.

    2.Область значений функции.

    3. y = 0, y> 0, y

    y =0, если x = 0.

    y

    4.Возрастания, убывания функции.

    Функция убывает при х .

    Построим график у=.

    Выделим его часть на отрезке . Заметим, что у наим. = 1 при х = 1, а у наиб. =3 при х = 9.

    Ответ: у наим. = 1, у наиб. =3

    6. Самостоятельная работа с самопроверкой по эталону

    Цель этапа: проверить своё умение применять новое учебное содержание в типовых условиях на основе сопоставления своего решения с эталоном для самопроверки.

    Организация учебного процесса на этапе 6:

    Учащиеся выполняют задание самостоятельно, проводят самопроверку по эталону, анализируют, исправляют ошибки.

    Построим график у=.

    С помощью графика найдите наименьшее и наибольшее значения функции на отрезке .

    7. Включение в систему знаний и повторение

    Цель этапа: тренировать навыки использования нового содержания совместно с ранее изученным: 2) повторить учебное содержание, которое потребуется на следующих уроках.

    Организация учебного процесса на этапе 7:

    Решите графически уравнение: = х – 6.

    Один ученик у доски остальные в тетрадях.

    8. Рефлексия деятельности

    Цель этапа:

    1) зафиксировать новое содержание, изученное на уроке;

    2) оценить собственную деятельность на уроке;

    3) поблагодарить одноклассников, которые помогли получить результат урока;

    4) зафиксировать неразрешённые затруднения как направления будущей учебной деятельности;

    5) обсудить и записать домашнее задание.

    Организация учебного процесса на этапе 8:

    – Ребята, какая цель стояла сегодня перед нами? (Изучить функцию у=, ее свойства и график).

    – Какие знания нам помогли в достижении цели? (Умение искать закономерности, умение читать графики.)

    – Проанализируйте свою деятельность на уроке. (Карточки с рефлексией)

    Домашнее задание

    п. 13 (до примера 2) 13.3, 13.4

    Решите графически уравнение.

    Муниципальное общеобразовательное учреждение

    средняя общеобразовательная школа №1

    ст. Брюховецкой

    муниципального образования Брюховецкий район

    Учитель математики

    Гученко Анжела Викторовна

    2014 год

    Функция у =
    , ее свойства и график

    Тип урока: изучение нового материала

    Цели урока:

    Задачи, решаемые на уроке:

      научить учащихся самостоятельно работать;

      высказывать предположения и догадки;

      уметь делать обобщение изучаемых факторов.

    Оборудование: доска, мел, мультимедийный проектор, раздаточный материал

    Хронометраж урока.

      Определение темы урока совместно с учащимися – 1мин.

      Определение целей и задач урока совместно с учащимися – 1мин.

      Актуализация знаний (фронтальный опрос) – 3мин.

      Устная работа — 3мин.

      Объяснение нового материала, построенное на создании проблемных ситуаций — 7мин.

      Физминутка – 2мин.

      Построение графика вместе с классом с оформлением построения в тетрадях и определением свойств функции, работа с учебником – 10мин.

      Закрепление полученных знаний и отработка навыков преобразований графиков – 9мин .

      Подведение итогов урока, установление обратной связи – 3мин.

      Задание на дом – 1мин.

    Итого 40 минут.

    Ход урока.

      Определение темы урока совместно с учащимися (1мин).

    Тема урока определяется учащимися при помощи наводящих вопросов:

      функция — работа, производимая органом, организмом в целом.

      функция — возможность, опция, умение программы или прибора.

      функция — обязанность, круг деятельности.

      функция персонажа в литературном произведении.

      функция — вид подпрограммы в информатике

      функция в математике — закон зависимости одной величины от другой.

      Определение целей и задач урока совместно с учащимися (1мин).

    Учитель при помощи учащихся формулирует и проговаривает цели и задачи данного урока.

      Актуализация знаний (фронтальный опрос – 3мин).

      Устная работа – 3 мин.

    Фронтальная работа.

    (А и В принадлежат, С нет)

      Объяснение нового материала (построено на создании проблемных ситуаций – 7мин).

    Проблемная ситуация: описать свойства неизвестной функции.

    Разбить класс на команды по 4-5 человек, раздать бланки для ответов на поставленные вопросы

    Бланк №1

      у=0, при х=?

      Область определения функции.

      Множество значений функции.

    На каждый вопрос отвечает один из представителей команды, остальные команды голосуют «за» или «против» сигнальными карточками и, если нужно, дополняют ответы одноклассников.

    Вместе с классом сделать вывод об области определения, множестве значений, нулях функции у=.

    Проблемная ситуация : попробовать построить график неизвестной функции (идет обсуждение в командах, поиск решения).

    С учителем вспоминается алгоритм построения графиков функций. Учащиеся командами пробуют изобразить график функции у= на бланках, затем обмениваются бланками друг с другом для само- и взаимопроверки.

    Физминутка (Клоунада)

      Построение графика вместе с классом с оформлением построения в тетрадях – 10мин.

    После общего обсуждения выполняется задание построения графика функции у= индивидуально каждым учеником в тетради. Учитель в это время оказывает дифференцированную помощь учащимся. После выполнения задания учащимися на доске показывается график функции и учащимся предлагается ответить на следующие вопросы:


    Вывод: вместе с учащимися сделать еще раз вывод о свойствах функции и прочитать их по учебнику:

      Закрепление полученных знаний и отработка навыков преобразования графика – 9мин.

    Учащиеся работают по своей карточке (по вариантам), затем меняются и проверяют друг друга. После на доске показываются графики, и учащиеся оценивают свою работу, сравнивая с доской.

    Карточка №1


    Карточка №2


    Вывод: о преобразованиях графика

    1) параллельный перенос вдоль оси ОУ

    2) сдвиг вдоль оси ОХ.

    9. Подведение итогов урока, установление обратной связи – 3мин.

    СЛАЙДЫ вставить пропущенные слова

      Область определения данной функции, все числа, кроме…(отрицательных).

      График функции расположен в … (I) четверти.

      При значении аргумента х = 0, значение… (функции) у = …(0).

      Наибольшее значение функции… (не существует), наименьшее значение — …(равно 0)

    10. Задание на дом (с комментариями – 1 мин).

    По учебнику — §13

    По задачнику – №13.3, №74 (повторение неполных квадратных уравнений)

    Цели урока:


    Оборудование:

      Компьютер, интерактивная доска, раздаточный материал.

      Презентация к уроку.

    ХОД УРОКА

    План урока.

      Вступительное слово учителя.

      Повторение ранее изученного материала.

      Изучение нового материала (групповая работа).

      Исследование функции. Свойства графика.

      Обсуждение графика (фронтальная работа).

      Игра в математические карты.

      Итоги урока.

    I. Актуализация опорных знаний.

    Приветствие учителя.

    Учитель :

    Зависимость одной переменной от другой называется функцией. До сих пор Вы изучили функции y = kx + b; y =к/х, у=х 2 . Сегодня мы продолжим изучение функций. На сегодняшнем уроке вы узнаете, как выглядит график функции квадратного корня, научитесь сами строить графики функций квадратного корня.

    Запишите тему урока ( слайд1).

    2. Повторение изученного материала.

    1. Как называются функции, задаваемые формулами:

    а) у=2х+3; б) у=5/х; в) у = -1/2х+4; г) у=2х; д) у=-6/х е) у =х 2 ?

    2. Что представляет собой их график? Как он расположен? Укажите область определения и область значения каждой из этих функций (на рис. изображены графики функций, заданные данными формулами, для каждой функции укажите её вид) ( слайд2).

    3. Что представляет из себя график каждой функции, как эти графики строятся?

    ( слайд3, строятся схематически графики функций).

    3. Изучение нового материала.

    Учитель :

    Итак, сегодня мы изучаем функцию
    и её график.

    Мы знаем, что графиком функции у=х 2 является парабола. Что будет графиком функции у=х 2 , если взять только х0 ? Является часть параболы — её правая ветвь. Построим теперь график функции
    .

    Повторим алгоритм построения графиков функций(слайд 4, с алгоритмом )

    Вопрос : Как вы считаете, глядя на аналитическую запись функции, можно сказать о том, какие значения х допустимы? (Да, х≥0 ). Так как выражение
    имеет смысл при всех х больших или равных 0.

    Учитель: В явлениях природы, в человеческой деятельности часто встречаются зависимости между двумя величинами. Каким графиком можно представить эту зависимость? (групповая работа )

    Класс разбивается на группы. Каждая группа получает задание: построить график функции
    на миллиметровой бумаге, выполняя все пункты алгоритма. Затем от каждой группы выходит представитель и показывает работу группы. (открывается слад 5, идет проверка, затем график строится в тетрадях)

    4. Исследование функции.(продолжается работа вгруппах)

    Учитель:

      найдите область определения функции;

      найдите область значения функции;

      определите промежутки убывания (возрастания) функции;

      у>0, у

    Записывамв результаты( слайд6).

    Учитель: Проведем анализ графика. Графиком функции является ветвь параболы.

    Вопрос : Скажите, вы встречали где-нибудь этот график раньше?

    Посмотрите на график и скажите, пересекает ли он прямую ОХ? (Нет) ОУ? (Нет) . Посмотрите на график и скажите, имеет ли график центр симметрии? Ось симметрии?

    Подведем итоги:


    Атеперь поверим, как усвоили новую тему и повторили пройденный материал. Игра в математические карты.(правила игры: каждой группе из 5 человек предлагается комплект карточек (25 карт). Каждый игрок получает по 5 карт, на которых написаны вопросы. Первый ученик дает одну из карт второму ученику, который должен ответить на вопрос из карточки. Если ученик отвечает на вопрос, то карта бита, если нет, то ученик забирает карту себе и предает ход и т.д. всего 5 ходов. Если у ученика не осталось карт, то оценка -5, осталась 1 карта-оценка 4, 2 карты – оценка 3, 3 карты – оценка- 2)

    5. Итоги урока. (выставляются оценки обучающимся по контрольным листам)

    Задание на дом.

      Изучить п.8.

      Решить №172, №179, №183.

      Подготовить сообщения на тему “Применение функции в различных областях науки и в литературе”.

    Рефлексия.

    Покажите свое настроение с помощью картинок на вашем столе.

    Сегодня урок

      Мне понравилось.

      Мне не понравилось.

      Материал урока я (понял, не понял).

    Как построить график \\[y = \\sqrt {x + 1} \\], сравнить его с родительским графиком и что такое домен и диапазон?

    Подсказка: Мы должны нарисовать график для данной функции, взяв различные значения \[x\], затем найти соответствующие значения \[y\], используя данную функцию, а затем нанести точки на графики \ [x\]-ось и \[y\]-ось. И мы должны найти домен и диапазон данной функции, мы знаем, что домен — это набор значений, для которых определена функция, а диапазон — это набор всех значений, которые функция \[y\] может достичь для этих значения \[x\], которые находятся в области определения функции \[y\].

    Полное пошаговое решение:
    Дан граф \[y = \sqrt {x + 1} \],
    Родительский граф данного графа \[\sqrt {x + 1} \] равно \[\sqrt x \].
    Домен функции — это когда бит внутри квадратного корня больше или равен нулю (иначе он не был бы определен в терминах действительных чисел). Мы можем узнать, когда это так, решив следующее неравенство0009 \[ \Rightarrow x + 1 — 1 \geqslant 0 — 1\],
    Упрощая, получаем,
    \[ \Rightarrow x \geqslant — 1\],
    Таким образом, искомый домен равен \[\left\{ {x|x \geqslant — 1} \right\}\],
    Теперь диапазон родительского графа,\[\sqrt x \] состоит из всех действительных положительных чисел и нулей, и перемещение влево ничего не меняет что, таким образом, областью действия этой функции также являются все положительные действительные числа и ноль, \[ R \].
    Теперь, чтобы построить график, мы должны взять любые случайные значения для \[x\], чтобы получить соответствующие значения для \[y\].
    Теперь возьмем \[x = — 1\], теперь подставив в функцию \[y = \sqrt {x + 1} \] получим,
    \[ \Rightarrow y = \sqrt { — 1 + 1} \] ,
    Теперь упрощая получаем,
    \[ \Rightarrow y = \sqrt 0 \],
    Итак снова упрощая получаем,
    \[ \Rightarrow y = 0\],
    Теперь берем \[x = 0\], теперь подставляя в функцию \[y = \sqrt {x + 1} \] получаем,
    \[ \Rightarrow y = \sqrt {0 + 1} \],
    Теперь упрощая получаем,
    \[ \Rightarrow y = \sqrt 1\],
    Итак, снова упрощая, получаем,
    \[ \Rightarrow y = 1\],
    Теперь возьмем \[x = 1\], теперь подставив в функцию \[y = \sqrt {x + 1} \] получим,
    \[ \Rightarrow y = \sqrt {1 + 1} \],
    Теперь, упрощая, получаем,
    \[ \Rightarrow y = \sqrt 2 \],
    Итак, снова упрощая, получаем,
    \[ \Rightarrow y = 1,414\],
    Теперь рисуем координаты на плоскости получаем,

    Родительский граф данного графа \[\sqrt {x + 1} \] равен \[\sqrt x \], а \[\sqrt {x + 1} \] — тот же график, но сдвинутый на 1 единицу влево.

    Онлайн калькулятор деление столбик: Онлайн калькулятор. Деление столбиком

    Калькулятор деления столбиком с остатком

    0
    AC +/- ÷
    7 8 9 ×
    4 5 6
    1 2 3 +
    0 00 , =

    Данный калькулятор выполнит деление двух целых чисел с остатком и отобразит запись деления столбиком.

    Введите целые неотрицательные числа

    ÷

    Как оформлять деление столбиком

    1. Делимое располагается слева от вертикальной черты, под ним следует записать промежуточное решение, а в конце остаток.
    2. Справа от вертикальной черты записывается делитель, под ним находится горизонтальная черта.
    3. Под горизонтальной чертой записывается частное .
    Как делить столбиком

    Приведем правила деления в столбик с остатком на примере. Разделим 453 на 2.

    Первое, что необходимо сделать – это определить неполное делимое. Неполное делимое должно быть меньше делителя. В нашем случае это число 4, выделим это число зеленым цвет

    Теперь определим сколько раз число 2 содержится в числе 4. Число 2 содержится в числе 4 два раза. Следовательно, умножаем 2 на 2 и вычитаем результат произведения из неполного делимого 4 – 4 = 0. В результате вычитания у нас получился ноль, поэтому сносим следующую цифру 5 из числа 453 и выделим ее зеленым цветом. Запишем 2 под горизонтальной чертой и выделим синем цветом.

    354_

    4

    50

    Далее снова определяем сколько раз делитель – число 2 содержится теперь уже в числе 5. Число 2 содержится в числе 5 два раза. Запишем еще оду двойку под горизонтальной чертой и выделим ее синем цветом. Умножим 2 на 2, получим 4 и вычитаем из 5 число 4.

    354_

    4

    50_

    4

    1

    Сносим последнее число 3, в результате имеем число 13. В числе 13 число 2 содержится 6 раз. Запишем число 6 под горизонтальной чертой. Умножим 6 на 2, получим 12. Вычтем из 13 число 12. 13 – 12 = 1. Остаток от деления = 1.

    354_

    4

    50_

    4

    31_

    21

    1

    Вам могут также быть полезны следующие сервисы
    Генератор Pdf с примерами
    Тренажёры решения примеров
    Тренажёр таблицы умножения
    Тренажер счета для дошкольников
    Тренажер счета на внимательность для дошкольников
    Тренажер решения примеров на сложение, вычитание, умножение, деление. Найди правильный ответ.
    Тренажер решения примеров с разными действиями
    Тренажёры решения столбиком
    Тренажёр сложения столбиком
    Тренажёр вычитания столбиком
    Тренажёр умножения столбиком
    Тренажёр деления столбиком с остатком
    Калькуляторы решения столбиком
    Калькулятор сложения, вычитания, умножения и деления столбиком
    Калькулятор деления столбиком с остатком
    Показать больше
    Калькуляторы систем счисления
    Калькулятор перевода чисел из арабских в римские и из римских в арабские
    Калькулятор перевода чисел в различные системы счисления
    Калькулятор сложения, вычитания, умножения и деления двоичных чисел
    Системы счисления теория
    N2 | Двоичная система счисления
    N3 | Троичная система счисления
    N4 | Четырехичная система счисления
    N5 | Пятеричная система счисления
    N6 | Шестеричная система счисления
    N7 | Семеричная система счисления
    N8 | Восьмеричная система счисления
    N9 | Девятеричная система счисления
    N11 | Одиннадцатиричная система счисления
    N12 | Двенадцатеричная система счисления
    N13 | Тринадцатеричная система счисления
    N14 | Четырнадцатеричная система счисления
    N15 | Пятнадцатеричная система счисления
    N16 | Шестнадцатеричная система счисления
    N17 | Семнадцатеричная система счисления
    N18 | Восемнадцатеричная система счисления
    N19 | Девятнадцатеричная система счисления
    N20 | Двадцатеричная система счисления
    N21 | Двадцатиодноричная система счисления
    N22 | Двадцатидвухричная система счисления
    N23 | Двадцатитрехричная система счисления
    N24 | Двадцатичетырехричная система счисления
    N25 | Двадцатипятеричная система счисления
    N26 | Двадцатишестеричная система счисления
    N27 | Двадцатисемеричная система счисления
    N28 | Двадцативосьмеричная система счисления
    N29 | Двадцатидевятиричная система счисления
    N30 | Тридцатиричная система счисления
    N31 | Тридцатиодноричная система счисления
    N32 | Тридцатидвухричная система счисления
    N33 | Тридцатитрехричная система счисления
    N34 | Тридцатичетырехричная система счисления
    N35 | Тридцатипятиричная система счисления
    N36 | Тридцатишестиричная система счисления
    Калькуляторы (Теория чисел)
    Калькулятор выражений
    Калькулятор упрощения выражений
    Калькулятор со скобками
    Калькулятор уравнений
    Калькулятор суммы
    Калькулятор пределов функций
    Калькулятор разложения числа на простые множители
    Калькулятор НОД и НОК
    Калькулятор НОД и НОК по алгоритму Евклида
    Калькулятор НОД и НОК для любого количества чисел
    Калькулятор делителей числа
    Представление многозначных чисел в виде суммы разрядных слагаемых
    Калькулятор деления числа в данном отношении
    Калькулятор процентов
    Калькулятор перевода числа с Е в десятичное
    Калькулятор экспоненциальной записи чисел
    Калькулятор нахождения факториала числа
    Калькулятор нахождения логарифма числа
    Калькулятор квадратных уравнений
    Калькулятор остатка от деления
    Калькулятор корней с решением
    Калькулятор нахождения периода десятичной дроби
    Калькулятор больших чисел
    Калькулятор округления числа
    Калькулятор свойств корней и степеней
    Калькулятор комплексных чисел
    Калькулятор среднего арифметического
    Калькулятор арифметической прогрессии
    Калькулятор геометрической прогрессии
    Калькулятор модуля числа
    Калькулятор абсолютной погрешности приближения
    Калькулятор абсолютной погрешности
    Калькулятор относительной погрешности
    Дроби
    Калькулятор интервальных повторений
    Учим дроби наглядно
    Калькулятор сокращения дробей
    Калькулятор преобразования неправильной дроби в смешанную
    Калькулятор преобразования смешанной дроби в неправильную
    Калькулятор сложения, вычитания, умножения и деления дробей
    Калькулятор возведения дроби в степень
    Калькулятор перевода десятичной дроби в обыкновенную
    Калькулятор перевода обыкновенной дроби в десятичную
    Калькулятор сравнения дробей
    Калькулятор приведения дробей к общему знаменателю
    Калькуляторы (тригонометрия)
    Калькулятор синуса угла
    Калькулятор косинуса угла
    Калькулятор тангенса угла
    Калькулятор котангенса угла
    Калькулятор секанса угла
    Калькулятор косеканса угла
    Калькулятор арксинуса угла
    Калькулятор арккосинуса угла
    Калькулятор арктангенса угла
    Калькулятор арккотангенса угла
    Калькулятор арксеканса угла
    Калькулятор арккосеканса угла
    Калькулятор нахождения наименьшего угла
    Калькулятор определения вида угла
    Калькулятор смежных углов
    Калькуляторы площади геометрических фигур
    Площадь квадрата
    Площадь прямоугольника
    КАЛЬКУЛЯТОРЫ ЗАДАЧ ПО ГЕОМЕТРИИ
    Калькуляторы (Комбинаторика)
    Калькулятор нахождения числа перестановок из n элементов
    Калькулятор нахождения числа сочетаний из n элементов
    Калькулятор нахождения числа размещений из n элементов
    Калькуляторы линейная алгебра и аналитическая геометрия
    Калькулятор сложения и вычитания матриц
    Калькулятор умножения матриц
    Калькулятор транспонирование матрицы
    Калькулятор нахождения определителя (детерминанта) матрицы
    Калькулятор нахождения обратной матрицы
    Длина отрезка. Онлайн калькулятор расстояния между точками
    Онлайн калькулятор нахождения координат вектора по двум точкам
    Калькулятор нахождения модуля (длины) вектора
    Калькулятор сложения и вычитания векторов
    Калькулятор скалярного произведения векторов через длину и косинус угла между векторами
    Калькулятор скалярного произведения векторов через координаты
    Калькулятор векторного произведения векторов через координаты
    Калькулятор смешанного произведения векторов
    Калькулятор умножения вектора на число
    Калькулятор нахождения угла между векторами
    Калькулятор проверки коллинеарности векторов
    Калькулятор проверки компланарности векторов
    Конвертеры величин
    Конвертер единиц длины
    Конвертер единиц скорости
    Конвертер единиц ускорения
    Цифры в текст
    Калькуляторы (физика)

    Механика

    Калькулятор вычисления скорости, времени и расстояния
    Калькулятор вычисления ускорения, скорости и перемещения
    Калькулятор вычисления времени движения
    Калькулятор времени
    Второй закон Ньютона. Калькулятор вычисления силы, массы и ускорения.
    Закон всемирного тяготения. Калькулятор вычисления силы притяжения, массы и расстояния.
    Импульс тела. Калькулятор вычисления импульса, массы и скорости
    Импульс силы. Калькулятор вычисления импульса, силы и времени действия силы.
    Вес тела. Калькулятор вычисления веса тела, массы и ускорения свободного падения

    Оптика

    Калькулятор отражения и преломления света

    Электричество и магнетизм

    Калькулятор Закона Ома
    Калькулятор Закона Кулона
    Калькулятор напряженности E электрического поля
    Калькулятор нахождения точечного электрического заряда Q
    Калькулятор нахождения силы F действующей на заряд q
    Калькулятор вычисления расстояния r от заряда q
    Калькулятор вычисления потенциальной энергии W заряда q
    Калькулятор вычисления потенциала φ электростатического поля
    Калькулятор вычисления электроемкости C проводника и сферы

    Конденсаторы

    Калькулятор вычисления электроемкости C плоского, цилиндрического и сферического конденсаторов
    Калькулятор вычисления напряженности E электрического поля плоского, цилиндрического и сферического конденсаторов
    Калькулятор вычисления напряжения U (разности потенциалов) плоского, цилиндрического и сферического конденсаторов
    Калькулятор вычисления расстояния d между пластинами в плоском конденсаторе
    Калькулятор вычисления площади пластины (обкладки) S в плоском конденсаторе
    Калькулятор вычисления энергии W заряженного конденсатора
    Калькулятор вычисления энергии W заряженного конденсатора. Для плоского, цилиндрического и сферического конденсаторов
    Калькулятор вычисления объемной плотности энергии w электрического поля для плоского, цилиндрического и сферического конденсаторов
    Калькуляторы по астрономии
    Вес тела на других планетах
    Ускорение свободного падения на планетах Солнечной системы и их спутниках
    Генераторы
    Генератор примеров по математике
    Генератор случайных чисел
    Генератор паролей

    Онлайн калькулятор деление в столбик




    Онлайн калькулятор деление в столбик

    Деление также происходит в столбик, но частное результат записывается ниже делителя и отделяется от него горизонтальной чертой. Код для вставки без рекламы с прямой ссылкой на сайт Код для вставки с рекламой без прямой ссылки на сайт Код для вставки: Скопируйте и вставьте этот код на свою страничку в то место, где хотите, чтобы отобразился калькулятор. Рассматриваем цифры делимого слева направо и определяем наименьшее число, но больше делителя. Обозначение в России, Казахстане, Киргизии, Франции, Бельгии, Испании, на Украине, в Белоруссии, Молдавии, Грузии, Таджикистане, Онлайн калькулятор деление в столбик, Монголии В России делитель располагается справа от делимого, отделяемого от него вертикальной чертой. Процесс деления столбиком американо-британский вариант числа 1 260 257 на число 37 Деление столбиком — стандартная процедура в арифметике, предназначенная для простых или сложных многозначных чисел за счёт разбивания деления на ряд более простых шагов. Научиться складывать числа столбиком самостоятельно можно в уроке. Одно из распространенных арифметических действий — деление в столбик. Полученный остаток должен быть меньше делителя. Выделяем первое неполное делимое и объясняем, какие разрядные единицы оно обозначает. Я владелец и автор этого сайта, мною написан весь теоретический материал, а также разработаны онлайн упражнения и калькуляторы, которыми Вы можете воспользоваться для изучения математики. Таким образом в нашем распоряжении как первое число теперь будет число 12. Умножаем это количество раз на делитель, произведение ставим под предыдущий результат и производим вычитание.

    Умножение и деление в столбик онлайн Наш онлайн калькулятор позволяет умножать и делить числа столбиком всего за несколько кликов мышкой. Все дело в том, что калькулятор при вводе очередного математического действия сложение, вычитание и др. Деление в столбик всегда пригодится — этим способом можно как найти остаток целочисленного деления, так и довести процесс деления до нужного знака после запятой. К остатку спускаем вниз следующую из оставшихся цифр делимого и продолжаем делить, пока не используем все цифры делимого, а при вычитании не получим 0. Научиться складывать числа столбиком самостоятельно можно в уроке.

    В переводе с греческого арифметика означает число. К остатку спускаем вниз следующую из оставшихся цифр делимого и продолжаем делить, пока не используем все цифры делимого, а при вычитании не получим 0. Полученный результат от деления называется частным. Деление в столбик онлайн позволяет выполнить деления двух чисел столбиком, и получить полностью расписанный процесс деления в столбик с получением целой части и остатка. Затем отмечаем, сколько раз делитель поместится в этом числе. Этот онлайн калькулятор поможет вам понять как разделить целые числа и десятичные дроби столбиком.

    Онлайн калькулятор деление в столбик

    Умножаем количество раз на делитель, полученный результат ставим под первым числом делимого, после чего вычитаем. Только самые простые функции: сложение, вычитание, умножение деление. Вместо этого делимое, делитель и частное в процессе нахождения располагаются в таблице. Деление также происходит в столбик, но частное результат записывается ниже делителя и отделяется от него горизонтальной чертой. Увеличиваем остаток, спустив к нему следующую цифру делимого. Похоже, вы используете блокировщик рекламы.

    Вы можете делить и умножать целые числа и проверять себя по подробному решению. Полученный остаток должен быть меньше делителя.


    Калькулятор синтетического деления

    Вычислите полиномы с помощью калькулятора синтетического деления, который позволит вам определить напоминание синтетического деления и частное полиномов с использованием метода синтетического деления. Он также находит нули знаменателя и коэффициента числителя.

    Вы хотите научиться применять шаги синтетического деления к полиномам? Здесь мы научим вас всему о делении полинома с помощью синтетического деления.

    Что такое синтетическое деление многочленов?

    Синтетическое деление представляет собой упрощенный способ деления полинома на другое полиномиальное выражение первой степени и обычно используется для определения нулей полинома.

    Этот метод выполняется с меньшими усилиями, чем расчет методом длинного деления. Биномиальное уравнение обычно используется в качестве делителя в методе синтетического деления.

    Как использовать метод синтетического деления?

    Если вы хотите разделить полиномы с помощью синтетического метода, вы должны делить его на старший коэффициент, который должен быть равен 1, или делить на линейное выражение.

    Требования к методу синтетического процесса:

    • Делитель данного полиномиального уравнения должен иметь степень один.
    • Старший коэффициент делителя также должен быть равен единице.

    Если делитель старшего коэффициента отличен от единицы, то синтетическое деление работать не будет.
    Основная техника выполнения синтетического деления:

    Опустить, умножить и сложить, умножить и сложить, умножить и сложить, ….

    Как делить полиномы с помощью синтетического деления?

    Вы можете выполнить синтетическое деление вручную, но это сложная задача, однако следующие шаги используются при делении с помощью калькулятора синтетического деления с шагами для синтетического процесса:

    Шаг 1:

    • Чтобы найти число для замены это в поле деления, нам нужно установить знаменатель как ноль.
    • Если какой-либо термин отсутствует, то запишите отсутствующий термин нулем и запишите числитель в порядке убывания.

    Шаг 2:

    • Уменьшите ведущий коэффициент, когда задача поставлена ​​идеально.

    Шаг 3:

    • Теперь замените результаты в следующем столбце, умножив число в поле деления на полученное число.

    Шаг 4:

    • Подставив два числа вместе, запишите результат внизу строки.

    Шаг 5:

    • Запишите окончательные результаты.
    • Переменные должны начинаться с одной степени меньше знаменателя и уменьшаться с каждым членом.

    Однако онлайн-калькулятор частного и остатка позволит вам разделить два числа, делимое и делитель, чтобы определить частное с остатком.

    Пример: 9{0}\\-2.0&7&0&4&8\\&&\\\hline&\end{array} \)

    Перенести ведущий коэффициент в нижнюю строку

    \( \begin{array}{c|rrrrr}-2.0&7&0&4&8 \\&&\\\hline&7\end{array} \)

    Теперь калькулятор синтетической подстановки умножает полученное значение на ноль знаменателей и помещает результат в следующий столбец
    $$ 7∗(−2. 0) = −14 $$
    \( \begin{array}{c|rrrrr}-2.0&7&0&4&8\\&&-14&\\\hline&7&\end{array} \)

    Добавьте вниз столбец
    $$ 0 + (−14) = −14 $$
    \( \begin{array}{c|rrrrr}-2.0&7&0&4&8\\&&-14&\\\hline&7&-14&\end{ array} \)

    Умножьте полученное значение на ноль знаменателей и поместите результат в следующий столбец
    $$ −14 ∗ (−2,0) = 28 $$

    \( \begin{array}{c |rrrrr}-2.0&7&0&4&8\\&&-14&28&\\\hline&7&-14&\end{array} \)
    Прибавьте вниз по столбцу
    $$ 4 + (28) = 32 $$

    \( \begin{array}{c|rrrrr}-2.0&7&0&4&8\\&&-14&28&\\\hline&7&-14&32&\end{array} \)
    Решатель синтетического деления умножает полученное значение на ноль знаменателей , и поместите результат в следующий столбец
    $$ 32 ∗ (−2.0 ) = −64 $$

    \( \begin{array}{c|rrrrr}-2.0&7&0&4&8\\&&-14&28&-64&\\\ hline&7&-14&32&\end{array} \)
    Теперь используйте полиномы калькулятора синтетического деления, чтобы сложить столбец 9{0} \\-2.0& 1&5&6 \\&&\\\hline&\end{array} \)
    Перенести ведущий коэффициент в нижнюю строку

    \( \begin{array}{c|rrrrr}-2. 0& 1&5&6 \\&&\\\hline&1\end{array} \)
    Калькулятор синтетической подстановки умножает полученное значение на ноль знаменателей и помещает результат в следующий столбец

    \( \begin{array}{c |rrrrr}-2.0&1&5&6\\&&-2&\\\hline&1&\end{array} \)
    Теперь калькулятор полиномиального синтетического деления складывает столбец
    $$ 5 + (-2) = 3 $$

    \( \begin{array}{c|rrrrr}-2.0&1&5&6\\&&-2&\\\hline&1&3&\end{array} \)
    Синтетический длинный Калькулятор деления умножает полученное значение на ноль знаменателей и заносит результат в следующий столбец.

    Здесь для длинного деления выражений алгебры вы также можете использовать наш другой полиномиальный калькулятор длинного деления.

    $$ 3 ∗ (−2.0) = -6 $$

    \( \begin{array}{c|rrrrr}-2.0&1&5&6\\&&-2&-6&\\\hline&1&3&\end{array} \) 92 + 5x + 6} {x + 2} $$
    $$ x + 3 + \frac {56} {x + 2} = x + 3 $$

    Как работает калькулятор синтетического деления с шагами?

    Онлайн-калькулятор синтетической подстановки делит многочлен на бином, используя синтетическое деление. Здесь мы пошагово объясним, как этот синтетический калькулятор помогает определить остаток и частное.

    Ввод:
    • Сначала подставьте полиномы в качестве делимого и делителя.
    • Нажмите кнопку «Рассчитать».

    Вывод:
    • Калькулятор синтетического деления полиномов находит коэффициенты числителя и ноль знаменателя.
    • Он также предоставляет частное и остаток полиномов.
    • Калькулятор деления полиномов показывает все шаги в виде четко определенной синтетической таблицы деления.

    Часто задаваемые вопросы:

    Почему важно синтетическое деление?

    Метод синтетического деления играет важную роль в эффективном и простом делении многочленов, поскольку он разбивает сложные уравнения на простые уравнения. Поэтому всякий раз, когда вы чувствуете препятствие в отношении того, как выполнять синтетическое деление с многочленами, попробуйте использовать этот лучший калькулятор синтетического деления теоремы об остатках, чтобы найти нули и устранить ваши трудности при работе со сложными алгебраическими выражениями.

    В чем польза синтетического метода?

    Синтетический метод обычно используется для определения нулей корней многочленов. Кроме того, вы также можете знать, как использовать синтетическое деление, чтобы решить, что такое частное.

    Всегда ли можно использовать синтетический метод?

    Если степень знаменателя не равна 1, то использовать синтетический метод нельзя. С другой стороны, если степень знаменателя больше 1, вам следует использовать длинное полиномиальное деление.

    Какие существуют типы полиномиального деления?

    Существует четыре различных типа полиномиального деления:

    • Полиномиальное деление на моном
    • Полиномиальное деление на бином
    • Многочлен Деление на другой многочлен
    • Одночлен Деление на другой одночлен

    Здесь давайте закодируем, что если вы хотите разложить эти полиномы на множители, вы можете разложить их на множители с помощью калькулятора деления полиномов за промежуток времени.

    Заключение:

    Воспользуйтесь онлайн-калькулятором длинного синтетического деления с шагами для деления двух разных многочленов на биномиальное, чтобы найти остаток синтетического деления и частное от деления. Синтетическое деление — это кратчайший путь деления многочленов для частного случая деления на линейный множитель, коэффициент которого равен единице.

    Ссылка:

    Форма источника Википедии: регулярное синтетическое деление, оценка полиномов по теореме об остатках, расширенное синтетическое деление, для немонических делителей, компактное расширенное синтетическое деление.

    Из источника Lumen Learning: два многочлена, использование синтетического деления для деления, деление многочлена второй степени, деление многочлена третьей степени, использование синтетического деления для деления многочлена четвертой степени.

    Из источника Purple Math: синтетическое деление полиномов, выполнение синтетического деления, этапы метода полиномиального синтетического деления, преимущества и недостатки метода синтетического деления.

    Калькулятор деления матрицы — символьный

    Поиск инструмента

    Найдите инструмент в dCode по ключевым словам:

    Просмотрите полный список инструментов dCode

    Matrix Division

    Инструмент для расчета матричного деления двух матриц (2×2, 3×3, 4×4, 5×5, …). Деление матрицы состоит из умножения на инвертированную матрицу.

    Результаты

    Matrix Division — dCode

    Теги: Matrix

    Share

    dCode и другие

    dCode бесплатен, а его инструменты являются ценным подспорьем в играх, математике, геокэшинге, головоломках и задачах, которые нужно решать каждый день!
    Предложение? обратная связь? Жук ? идея ? Запись в dCode !

    Раздел матрицы

    Деление 2-х матриц

    Matrix M1

    Загрузка…
    (если это сообщение не исчезнет, ​​попробуйте обновить эту страницу)

    Matrix M2

    Загрузка…
    (если это сообщение не исчезнет, ​​попробуйте обновить эту страницу)

    Деление матрицы на скаляр (число)

    Matrix M

    Загрузка. {-1} $$ 9{-1} $$ полученный результат является результатом деления матрицы.

    Пример: Деление матриц 2×2 $$ \begin{bmatrix} 0 & 1 \\ 2 & 3 \end{bmatrix} / \begin{bmatrix} 1 & 2 \\ 3 & 4 \end{bmatrix} = \begin{bmatrix} 0 и 1 \\ 2 и 3 \end{bmatrix} . \left( \frac{1}{2} \begin{bmatrix} -4 & 2 \\ 3 & -1 \end{bmatrix} \right) = \frac{1}{2} \begin{bmatrix} 3 & -1 \\ 1 & 1 \end{bmatrix} $$

    Для выполнения деления необходимо соблюдать правила умножения матриц: $M_1$ должно иметь такое же количество $n$ столбцов, как и количество строк матрицы $M_2$. Более того, чтобы быть обратимой матрицей, матрица $M_2$ должна быть квадратной и, следовательно, иметь размер $n\times n$.

    Как разделить матрицу на скаляр?

    Делением матрицы $M=[a_{ij}]$ на скаляр $\lambda$ является матрица того же размера, что и $M$ (исходная матрица), причем каждый элемент матрицы делится на $ \лямбда $.

    $$ \frac{M}{\lambda} = [ a_{ij} / \lambda ] $$

    Пример: $$ \begin{bmatrix} 0 & 2 \\ 4 & 6 \end{bmatrix } / 2 = \begin{bmatrix} 0 & 1 \\ 2 & 3 \end{bmatrix} $$

    Исходный код

    dCode сохраняет право собственности на исходный код Matrix Division. За исключением явной лицензии с открытым исходным кодом (указано Creative Commons/бесплатно), алгоритма «Matrix Division», апплета или фрагмента (конвертер, решатель, шифрование/дешифрование, кодирование/декодирование, шифрование/дешифрование, транслятор) или «Matrix Division» функции (вычисление, преобразование, решение, расшифровка/шифрование, расшифровка/шифрование, декодирование/кодирование, перевод), написанные на любом информационном языке (Python, Java, PHP, C#, Javascript, Matlab и т. д.) и загрузка всех данных, скрипт, или доступ к API для «Matrix Division» не является общедоступным, то же самое для автономного использования на ПК, мобильных устройствах, планшетах, iPhone или в приложении для Android!
    Напоминание: dCode можно использовать бесплатно.

    Cite dCode

    Копирование и вставка страницы «Matrix Division» или любых ее результатов разрешено, если вы цитируете dCode!
    Бесплатный экспорт результатов в виде файла .csv или .txt осуществляется нажатием значка export
    Ссылка в качестве источника (библиография):
    Matrix Division на dCode.

    Упростить выражение онлайн с векторами: Упрощение выражений · Калькулятор Онлайн

    Сложение и вычитание векторов. Умножение вектора на число 10 класс онлайн-подготовка на Ростелеком Лицей

     

     

    Отметим, что сложение векторов производится аналогично планиметрии, только все действия выполняются в пространстве.

     

    Итак, пусть заданы два произвольных вектора в пространстве (рис. 1):

    Рис. 1. Произвольные векторы в пространстве

    Определим, что же называется суммой двух этих векторов.

    Точно так же, как в планиметрии, из любой удобной точки, назовем ее точкой А, можно единственным образом отложить вектор, равный вектору . Напомним, что заданные векторы, как и любые другие, свободны, важно лишь направление и длина, сам вектор можно параллельно переносить в любое место как на плоскости, так и в пространстве. Так, мы получили вектор  – в результате действия вектора  точка А переместилась в точку В. Теперь из точки В откладываем единственно возможным образом вектор , получаем вектор  – так, в результате действия вектора  точка В переместилась в точку С. В результате точка А переместилась в точку С, получен вектор , который и называется суммой векторов  и  (рис. 2).

    Рис. 2. Сумма двух векторов в пространстве

    Так, получено правило треугольника для сложения векторов в пространстве.

    Правило треугольника

    Из любой точки пространства (точка А) откладываем первый вектор, из конца первого вектора (точка В) откладываем второй вектор и получаем точку С. Вектор, соединяющий начало первого вектора (точка А) и конец второго (точка С), и будет результирующим.

    Отметим, что результат сложения векторов не зависит от выбора начальной точки, существует соответствующая теорема, которая это доказывает на основании того, что из точки можно отложить вектор, равный заданному, единственным образом.

    Определение

    Разностью двух векторов называется такой третий вектор, который, будучи сложенным со вторым вектором, даст первый вектор.

    Введем разность векторов  и , для этого сложим вектор  с противоположным вектором :

    Итак, из произвольной точки А откладываем вектор , получаем точку В. Чтобы получить вектор  мы строим вектор, равный вектору  по длине, но противонаправленный. Полученный вектор откладываем из точки В – получаем точку D. Вектор  и будет искомым вектором разности.

    Проиллюстрируем (рис. 3):

    Рис. 3. Вычитание двух векторов в пространстве

    Построим на заданных векторах  и  параллелограмм (рис. 4):

    Рис. 4. Параллелограмм на двух заданных векторах

    Т. к. вектор ; аналогично .

    По правилу треугольника:

    Так, одна из диагоналей параллелограмма, построенного на двух векторах, соответствует сумме этих векторов.

    Рассмотрим разность векторов. По правилу треугольника:

    .

    Так, вторая диагональ параллелограмма, построенного на двух векторах, соответствует разности этих векторов.

    Для сложения и вычитания нескольких векторов применяется правило многоугольника. Пусть заданы векторы  и :

    Рис. 5. Три вектора в пространстве

    Необходимо построить вектор .

    Видим, что перед некоторыми векторами стоят численные множители. Напомним, что при умножении вектора на число получаем сонаправленный вектор, длина которого – это длина исходного вектора, умноженная на заданное число. Получим векторы  и . Вектор  сонаправлен с вектором , длина его в три раза больше. Вектор  противонаправлен вектору , длина его в два раза больше. Проиллюстрируем (рис. 6):

    Рис. 6. Умножение вектора на число

    Приступаем к сложению. Из произвольной точки А откладываем полученный вектор  – получаем точку В. Из точки В откладываем вектор  – получаем точку С. Из точки С откладываем вектор  – получаем точку D. Согласно правилу многоугольника, вектор  соответствует искомому вектору :

    Рис. 7. Сложение векторов по правилу многоугольника

    Задача 1:

    Задан тетраэдр ABCD (рисунок 8). Доказать:

     

    Рис. 8. Тетраэдр, задача 1

    Решение:

    По правилу треугольника:

    Аналогично:

    , ч. т. д.

    По правилу треугольника:

    Аналогично: , ч. т. д.

    Задача 2

    Упростить выражение:

    Рассмотрим отдельно сумму двух векторов: , ее значение очевидно:

    Проиллюстрируем (рис. 9):

    Рис. 9. Сумма двух векторов

    Теперь сократим противоположные векторы:

    Можно было сразу заметить:

    .

    В результате упрощения получено:

    .

    Итак, мы ввели операции сложения и вычитания векторов, умножения вектора на число в стереометрии, отметили, что операции аналогичны таким же для планиметрии. Кроме того, решили несколько задач, базирующихся на описанных операциях.

     

    Список литературы

    1. Геометрия. 10–11 класс: учебник для учащихся общеобразовательных учреждений (базовый и профильный уровни) / И. М. Смирнова, В. А. Смирнов. – 5-е издание, исправленное и дополненное – М.: Мнемозина, 2008. – 288 с.: ил.
    2. Геометрия. 10–11 класс: учебник для общеобразовательных учебных заведений / Шарыгин И. Ф. – М.: Дрофа, 1999. – 208 с.: ил.
    3. Геометрия. 10 класс: учебник для общеобразовательных учреждений с углубленным и профильным изучением математики /Е. В. Потоскуев, Л. И. Звалич. – 6-е издание, стереотип. – М.: Дрофа, 2008. – 233 с.: ил.

     

    Дополнительные рекомендованные ссылки на ресурсы сети Интернет

    1. Ru.onlinemschool.com (Иточник).
    2. Emomi.com (Источник).
    3. Cleverstudents.ru (Источник).

     

    Домашнее задание

    Задача 1: задан параллелепипед (рисунок 10). Доказать:

    1.

    2.

    3.

    Рис. 10. Параллелепипед

    Задача 2: упростить выражение:

    Задача 3: построить вектор , если векторы  и  заданы на рисунке 11:

    Рис. 11. Векторы, задача 3

     

    §1. Определение вектора. Операции над векторами — ЗФТШ, МФТИ

    1. Основные определения

    Удивительно, но с векторными величинами разной природы (перемещением, скоростью, силой, импульсом и др.) можно работать в значительной мере единообразно — как с геометрическими объектами — геометрическими векторами, или просто векторами, хотя есть и нюансы (см. ниже).

    Стрелка компаса — не вектор, т. к. для неё нет таких операций.

    Мы будем рассматривать векторы на плоскости и в соответствии со сложившейся традицией обозначать их латинскими буквами со стрелками наверху, например: `vec v`, `vec F`, `vec a`, `vec b` и т. п. Часто в целях экономии используют упрощённое обозначение — букву с чертой, например, `bar v` или `bar F`.

    Одну из граничных точек вектора называют его началом, а другую — концом. Направление вектора задаётся от начала к концу, причём на чертеже конец вектора отмечают стрелкой. Начало вектора называют также точкой его приложения. Если точка `A` является нача­лом вектора `vec a`, то мы будем говорить, что вектор `vec a` приложен в точке `A` (рис. 2).

    Число, выражающее длину направленного отрезка, называют модулем вектора и обозначают той же буквой, что и сам вектор, но без стрелки наверху, например: модулем вектора `vec v` является число `v`.  Часто для обозначения модуля вектора прибегают к помощи знака абсолютной величины и пишут, например, `|vec v|` или `|vec F|`.

    Вектор называется нулевым, если его начало и конец совпадают. Нулевой вектор не имеет определённого направления и его длина (модуль) равна нулю.

    Векторы называются коллинеарными, если они лежат либо на одной прямой, либо на параллельных прямых. Так, например, на рис. 3 векторы `vec a`, `vec b` и `vec c` коллинеарны. 

    На рис. 4 слева изображены неравные векторы `vec a` и `vec f`, `vec g` и `vec h`, а справа — равные векторы `vec p` и `vec q`. Точка приложения геометрического вектора `vec a` может быть выбрана произвольно. Мы не различаем двух равных векторов, имеющих разные точки приложения и получающихся один из другого параллельным переносом. В соответствии с этим векторы, изучаемые в геометрии, называют свободными (они определены с точностью до точки приложения).

    В физике точка приложения вектора иногда имеет  принципиальное значение. Достаточно вспомнить рычаг: две равные по модулю силы, направленные в одну и ту же сторону, производят на рычаг разное действие, если плечи сил не равны друг другу. И всё же сами силы равны друг другу! Бывают и случаи, когда вектору трудно приписать конкретную точку приложения. Например, если одна система отсчёта движется  относительно другой со скоростью `vec v`, то какой точке  приписать эту скорость?  Всем точкам движущейся системы!

    2. Сложение двух векторов.

    Пусть даны два произвольных вектора `vec a` и `vec b` (рис. 5а). 

    Для нахождения их суммы нужно перенести вектор `vec b` параллельно самому себе так, чтобы его начало совпало с концом вектора `vec a`. Тогда вектор, проведённый из начала вектора `vec a` в конец перенесённого вектора `vec b`, и будет являться суммой `vec a` и `vec b`. На рис. 5б — это вектор `vec c`.

    Описанное правило есть просто определение суммы векторов. Как и в случае с числами, сумма векторов не зависит от порядка слагаемых, и поэтому можно записать

    Приведённое выше правило геометрического сложения векторов называется правилом треугольника.

    Сумма векторов может быть найдена и по правилу параллелограмма. В этом случае параллельным переносом нужно совместить начала векторов `vec a` и `vec b` и построить на них, как на сторонах,  параллелограмм. Тогда сумма `vec a` и `vec b` будет представлять собой диагональ этого параллелограмма, конкретно — суммой `vec a` и `vec b` будет вектор, начало которого совпадает с общим началом векторов `vec a` и `vec b` конец расположен в противоположной вершине параллелограмма, а длина равна длине указанной диагонали (рис. 5в).

    Оба способа сложения дают идентичный результат и одинаково часто применяются на практике. Когда речь идёт о нахождении суммы трёх и более векторов, часто последовательно используют  правило  треугольника. Поясним сказанное.

    3. Сложение трёх и более векторов. 

    Пусть нужно сложить три вектора `vec a`, `vec b` и `vec d` (рис. 6). 

    Для этого  по правилу треугольника сначала находится сумма любых двух векторов, например `vec a` и `vec b`, потом полученный вектор `vec c = vec a + vec b` по тому же правилу складывается с третьим  вектором  `vec d`. Тогда  полученный  вектор `vec f = vec c + vec d` и  будет представлять собой сумму  трёх  векторов `vec a`, `vec b` и `vec d`: `vec f = vec a + vec b + vec d`. Как и в случае с двумя векторами, порядок слагаемых не влияет на конечный результат.

    Чтобы упростить процесс сложения трёх и более векторов, обычно не находят промежуточные суммы типа `vec c = vec a + vec b`, а применяют правило многоугольника: параллельными переносами из конца первого вектора откладывают второй, из конца второго — откладывают третий, из конца третьего  — четвёртый  и  т.  д. 

    Так,  на рис. 7 вектор  `vec g`  представляет собой сумму векторов `vec a`, `vec b`, `vec d`, `vec e`,  найденную по правилу многоугольника: `vec g = vec a + vec b + vec d + vec e`.

    В последнем равенстве мы встречаемся с умножением вектора на скаляр. Поясним эту процедуру.

    4. Умножение вектора на скаляр. 

    Произведением вектора `vec a` на число `k` называют новый вектор `vec b = k vec a`, коллинеарный вектору `vec a`, направленный в ту же сторону, что и вектор `vec a`, если `k > 0`, и в противоположную сторону, если `k < 0`, а модуль `b` равен

    где `|k|` — абсолютная величина числа `k`.  

    Если два вектора коллинеарны, то они отличаются только скалярным множителем. Наоборот, если два вектора отличаются только ска­лярным множителем, не равным  нулю, то они коллинеарны.      

    В случае, когда `k = 0` или `vec a = 0`, произведение `k vec a` представляет собой нулевой  вектор,  направление которого не определено.

    Если `k = 1`, то согласно (2) `vec b = vec a` и векторы `vec a` и `vec b` равны (рис. 8а).

    При `k = — 1` получим `vec b = — vec a`. Вектор `- vec a` имеет модуль, равный модулю вектора `vec a`, но направлен в противоположную сторону (рис. 8б).

    Импульс тела `vec p = m vec v` коллинеарен вектору скорости и направлен с ней в одну сторону, т. к. массы всех тел положительны. Чуть ранее говорилось об аддитивности импульса. Если система состоит из материальных точек с массами `m_1`, `m_2`, `m_3`, `…`, которые в некоторый момент времени имели скорости `vec(v_1)`, `vec(v_2)`, `vec(v_3)`, `…`, т. е. имели импульсы `vec(p_1) = m_1 vec(v_1)`, `vec(p_2) = m_2 vec(v_2)`, `vec(p_3) = m_3 vec(v_3)`, `…`, то вся система в этот момент обладает импульсом  

    При этом каждое из слагаемых здесь должно быть найдено по правилу умножения вектора (скорости данной частицы) на скаляр (её массу), а затем все эти векторы должны быть сложены, например, по правилу многоугольника.

    5. Разность двух векторов.

    Вычесть из вектора `vec a` вектор `vec b` означает прибавить к вектору `vec a` вектор   `- vec b`:

    см. рис.  9а, 9б.

    Упрощение выражений | nool

    Перейти к основному содержанию

    Домашняя страница Технологического института Онтарио

    nool

    Алгебраические выражения иногда могут выглядеть беспорядочно, поскольку они содержат не только числа, но и буквы алфавита. Давайте подробнее рассмотрим, как мы можем упростить эти выражения.

    Чтобы упростить алгебраическое выражение, мы должны собрать одинаковые термины. При упрощении алгебраического выражения находится эквивалентное выражение, которое проще исходного. Обычно это означает, что упрощенное выражение меньше исходного выражения. Существует множество различных видов алгебраических выражений, поэтому стандартной процедуры для их упрощения не существует. Вот список шагов, которым нужно следовать.

    • Подготовьте алгебраическое выражение для упрощения (например, путем расширения).
    • Определите и сгруппируйте похожие термины.
    • Объедините похожие термины.

    Пример: Упростите выражение 5x + 3y -9z -8x + 6y.

    Решение:

    Выражение не нужно готовить, поэтому сначала определите и сгруппируйте одинаковые члены:

    (5x — 8x) + (3y + 6y) — 9z

    Затем объедините одинаковые члены:

    -3x + 9y — 9z

    Пример: Упростите выражение 4(5a — 4b) -7(6a + 2b).

    Решение:

    Сначала подготовьте выражение для упрощения (расширьте): 20a — 16b — 42a- 14b

    Затем определите и сгруппируйте одинаковые термины: (20a — 42a) + (-16b — 14b)

    Наконец, объедините одинаковые термины: -22a — 20b

    Важно понимать, что не все алгебраические выражения можно упростить. Например, выражение 56a — 8b + 7c -5 не может быть упрощено дальше, так как в выражении нет одинаковых членов.

    Закончим еще одним примером выражения, в котором есть произведения и частные простых множителей, включающих степени с одним и тем же основанием. Их можно легко упростить, добавляя и вычитая индексы степеней (используя экспоненциальные законы).

    Пример: Упростите выражение 004 Выражение не нужно подготавливать, поэтому объедините похожие термины: 12w 3 x 5 /yz

    Пример — объединение рациональных выражений:

    Как избежать распространенных математических ошибок при упрощении:

    90 004 Упрощение с экспонентами:

    Алгебраическое сложение и вычитание векторов

    Введение

     

    В этой статье мы рассмотрим вектор. Векторы, в отличие от простых чисел (скаляров), которые имеют только величину, имеют как величину (длину), так и направление. Мы изучим, как представлять векторные величины, а также как их складывать и вычитать.

     

    Ключевые термины

     

  • Скаляр
  • Вектор
  • Нулевой вектор
  •  

    Цели

     

     

  • Количественное определение векторов используя систему координат
  • Сложение и вычитание векторов графически и алгебраически

     

  •  

    Отдельные числа, то есть значения, имеющие только (положительную или отрицательную) величину, называются скалярами. Числа 0, –3, π, i, 1.3, e, и т. д. — все это примеры скаляров. Другой тип значения, который часто используется в математике, — это вектор. Вектор — это величина, имеющая как величину , так и направление . В этой статье мы рассмотрим некоторые математические характеристики векторов. Векторы имеют широкое применение, например, в физике.

    Введение в векторы

    Чтобы понять разницу между скаляром и вектором, полезно подумать о физических примерах. Возьмем, к примеру, температуру. Вы можете использовать термометр для измерения температуры воздуха в разных местах. В каждом случае вы получаете некоторое число (и единицу измерения), скажем, 65°F. Это величина, но с ней не связано никакого направления; таким образом, это скалярная величина. Теперь рассмотрим измерения ветра в тех же местах. Когда вы измеряете ветер, вы, скорее всего, измеряете и скорость, и направление. Таким образом, ваши измерения ветра составляют вектор. Мы могли бы выразить этот вектор как стрелку, направленную в направлении ветра, причем длина стрелки пропорциональна скорости ветра. Ниже приведена иллюстрация двух измерений ветра, сделанных в разных точках; стрелки представляют векторы, связанные с этими измерениями.

    Векторы имеют величину и направление, но не имеют назначенного местоположения как такового. То есть, пока сохраняется направление и длина «стрелки», мы можем перемещать ее куда угодно, не меняя ее. Это важная характеристика, которая позволит нам широко работать с векторами.

     

    A Представление векторов

    Наша первая задача — найти способ четкого и последовательного представления векторов. Графически это просто: поскольку мы можем перемещать вектор куда угодно, давайте всегда располагать «хвост» вектора в начале координат плоскости. (Обратите внимание, что «голова» и «хвост» вектора определяются, как показано ниже.)

     

     

     

    Теперь, поместив хвост вектора в начало координат (помните, что мы можем перемещать вектор куда угодно, пока сохраняем его направление и длину), мы можем количественно определить его как координаты голова. Пример показан ниже для вектора v . (Обратите внимание, что для того, чтобы отличить символы, представляющие векторы, от символов, представляющих скаляры, мы используем жирный шрифт. Другой распространенный метод — использовать маленькую стрелку над символом: например, вектор.)

     

    Хотите узнать больше? Почему бы не пройти онлайн-курс Precalculus?

    Таким образом, вектор v — это просто координаты точки (2, 3). Обратите внимание, что все векторы, показанные ниже, равны (2, 3) — наше соглашение состоит в том, что вектор описывается координатами точки в его начале только , когда его конец расположен в начале координат.

     

     

     

    Хотя мы показали вектор только в двух измерениях, этот подход можно обобщить на любое количество измерений. Например, в трех измерениях вектор будет иметь вид ( x, y, z ). Все свойства двумерных векторов можно легко распространить на три измерения.

    Но как нам «переместить» вектор с числовой точки зрения? Например, скажем, вектор v имеет голову в (3, 2) и хвост в (1, 4).

     

     

     


    Ответ заключается в перемещении (или перемещении) головы и хвоста на эквивалентное расстояние и в одном направлении. Этот перевод должен привести к перемещению хвоста вектора в начало координат — простой процесс, который включает вычитание каждой хвостовой координаты из самой себя. В приведенном выше примере результат равен (3 – 3, 2 – 2) = (0, 0). Чтобы переместить голову, аналогичным образом вычтите координаты хвоста из координат головы — это удовлетворяет нашему критерию, согласно которому перемещение имеет фиксированное расстояние и направление. Таким образом, голову следует двигать следующим образом: (1 – 3, 4 – 2) = (–2, 2). Таким образом, в общем случае, чтобы найти значение произвольно расположенного вектора, нужно вычесть координаты хвоста из координат головы. Этот процесс проиллюстрирован ниже.

     

    Обратите внимание, что вектор (0, 0), иногда называемый нулевым вектором , имеет длину 0, но не имеет определенного направления. (То есть независимо от того, какое направление вы выберете, нулевой вектор один и тот же.)

    Практическая задача: Определите значение каждого вектора, показанного на графике ниже.

     

    Решение: В каждом случае можно найти координатное выражение для вектора, вычитая хвостовые координаты из соответствующих головных координат. Это работает, даже если хвост находится в начале координат, который имеет координаты (0, 0). Но если хвост находится в начале координат, вектор также просто равен координатам головы. Если это вам поможет, перерисуйте векторы так, чтобы хвосты располагались в начале координат.

    a = (–1, 4)

     

    b = (–3, –3)

      

    c = (3 – 3, 2 – 0) = (0, 2)

     

    d = (3 – 2, –4 – [–1]) = (1, –3)

     

    Сложение и вычитание векторов 9 0003


    Как и в случае со скалярами, мы можем добавить и вычесть векторы. Процесс аналогичен, но с одной или двумя оговорками. Чтобы сложить или вычесть два вектора a и b добавить или вычесть соответствующие координаты вектора. То есть там, где a и b определяются следующим образом, здесь действуют правила сложения и вычитания.

     


    Обратите внимание, что, как и в случае со скалярами, сложение векторов является коммутативным , а вычитание — нет. Графически мы добавляем два вектора a и b , позиционируя хвост b в начале a , а затем создайте новый вектор, начиная с хвоста a и заканчивая головой b . Координаты этого нового вектора определяются так же, как и раньше: размещением его хвоста в начале координат. Этот процесс проиллюстрирован ниже для векторов a = (4, 1) и b = (-1, 2).

     

     

     

    Обратите внимание, что

     

     

     

     

    Вычитание векторов происходит в основном по той же процедуре, что и сложение, за исключением того, что вычитаемый вектор «обращается» по направлению. Рассмотрим те же векторы a и b , что и выше, за исключением того, что мы будем вычислять a b. (Обратите внимание, что это то же самое, что и , где – b имеет ту же длину, что и b , но противоположно по направлению.)

     

    Практическая задача: Выполните следующие векторные операции.

     

     

    а. (3, 2) — (4, 5) б. (-1, 5) + (10, -6) в. (-1, 0) — (0, 0)

     

     

    Решение: В каждом случае сложите или вычтите соответствующие координаты, чтобы найти результат.